Location via proxy:   [ UP ]  
[Report a bug]   [Manage cookies]                

Probability Concepts and Random Variable - SMTA1402: Unit - I

Download as pdf or txt
Download as pdf or txt
You are on page 1of 105

SCHOOL OF SCIENCE AND HUMANITIES

DEPARTMENT OF MATHEMATICS

UNIT – I - Probability Concepts and Random Variable – SMTA1402


Sathyabama Institute of Science and Technology
SMTA1402 - Probability and Statistics

Unit-1 Probability Concepts and Random Variable

Random Experiment

An experiment whose outcome or result can be predicted with


certainty is called a Deterministic experiment.
Although all possible outcomes of an experiment may be known in
advance the outcome of a particular performance of the experiment cannot
be predicted owing to a number of unknown causes. Such an experiment is
called a Random experiment.
(e.g.) Whenever a fair dice is thrown, it is known that any of the 6 possible
outcomes will occur, but it cannot be predicted what exactly the outcome
will be.

Sample Space

The set of all possible outcomes which are assumed equally likely.

Event

A sub-set of S consisting of possible outcomes.

Mathematical definition of Probability

Let S be the sample space and A be an event associated with a random


experiment. Let n(S) and n(A) be the number of elements of S and A. then
the probability of event A occurring is denoted as P(A), is denoted by
n( A)
P( A) 
n( S )
Note: 1. It is obvious that 0  P(A)  1.
2. If A is an impossible event, P(A) = 0.
3. If A is a certain event , P(A) = 1.

A set of events is said to be mutually exclusive if the occurrence of any one


them excludes the occurrence of the others. That is, set of the events does
not occur simultaneously,
P(A1  A2  A3 ….. An,….. ) = 0 A set of events is said to be mutually
exclusive if the occurrence of any one them excludes the occurrence of the
others. That is, set of the events does not occur simultaneously,
P(A1  A2  A3 ….. An,….. ) = 0

Axiomatic definition of Probability

Let S be the sample space and A be an event associated with a random


experiment. Then the probability of the event A, P(A) is defined as a real
number satisfying the following axioms.
1. 0  P(A)  1
2. P(S) = 1
3. If A and B are mutually exclusive events, P(A  B) = P(A) + P(B)
and
4. If A1, A2 A3,….., An,….. are mutually exclusive events,
P(A1  A2  A3 ….. An,….. ) = P(A1) + P(A2) + P(A3) + ….. +
P(An)…..

Page no 1
Sathyabama Institute of Science and Technology
Important Theorems

Theorem 1: Probability of impossible event is zero.

Proof: Let S be sample space (certain events) and  be the impossible


event.
Certain events and impossible events are mutually exclusive.
P(S  ) = P(S) + P() (Axiom 3)
S=S
P(S) = P(S) + P()
P() = 0, hence the result.

Theorem 2: If A is the complementary event of A, P( A )  1  P( A)  1 .

Proof: Let A be the occurrence of the event


A be the non-occurrence of the event .
Occurrence and non-occurrence of the event are mutually exclusive.
P( A  A )  P( A)  P( A )
A A  S  P( A  A )  P( S )  1
 1  P( A)  P( A )
P( A )  1  P( A)  1 .

Theorem 3: (Addition theorem)


If A and B are any 2 events,
P(A  B) = P(A) + P(B)  P(A  B)  P(A) + P(B).

Proof: We know, A  AB  AB and B  A B  AB


 P( A)  P( AB )  P( AB) and P( B)  P( A B)  P( AB) (Axiom 3)
P( A)  P( B)  P( AB )  P( AB)  P( A B)  P( AB)
 P( A  B)  P( A  B)
P(A  B) = P(A) + P(B)  P(A  B)  P(A) + P(B).

Note: The theorem can be extended to any 3 events, A,B and C


P(A  B  C) = P(A) + P(B) +P(C)  P(A  B)  P(B  C)  P(C  A) +
P(A  B  C)

Theorem 4: If B  A, P(B)  P(A).

Proof: A and AB are mutually exclusive events such that B  AB  A


 P( B  AB )  P( A)
P( B)  P( AB )  P( A) (Axiom 3)
P ( B )  P ( A)

Conditional Probability

The conditional probability of an event B, assuming that the event A has


happened, is denoted by P(B/A) and defined as
P( A  B)
P( B / A)  , provided P(A)  0
P( A)

Page no 2
Sathyabama Institute of Science and Technology
Product theorem of probability

Rewriting the definition of conditional probability, We get


P( A  B)  P( A) P( A / B)
The product theorem can be extended to 3 events, A, B and C as follows:
P( A  B  C )  P( A) P( B / A) P(C / A  B)
Note: 1. If A  B, P(B/A) = 1, since A  B = A.
P( B)
2. If B  A, P(B/A)  P(B), since A  B = B, and  P( B),
P( A)
As P(A)  P(S) = 1.
3. If A and B are mutually exclusive events, P(B/A) = 0, since P(A
 B) = 0.
4. If P(A) > P(B), P(A/B) > P(B/A).
5. If A1  A2, P(A1/B)  P(A2/B).

Independent Events

A set of events is said to be independent if the occurrence of any one


of them does not depend on the occurrence or non-occurrence of the others.
If the two events A and B are independent, the product theorem takes
the form P(A  B) = P(A)  P(B), Conversely, if P(A  B) = P(A)  P(B),
the events are said to be independent (pair wise independent).
The product theorem can be extended to any number of independent
events, If A1 A2 A3 ….. An are n independent events, then
P(A1  A2  A3 ….. An) = P(A1)  P(A2 ) P(A3 )….. P(An)

Theorem 4:

If the events A and B are independent, the events A and B are also
independent.

Proof:
The events A  B and A  B are mutually exclusive such that (A  B) 
( A  B) = B
 P(A  B) + P( A  B) = P(B)
P( A  B) = P(B)  P(A  B)
= P(B)  P(A) P(B) (A and B are
independent)
= P(B) [1  P(A)]
= P( A ) P(B).
Theorem 5:
If the events A and B are independent, the events A and B are also
independent.

Proof:
P( A  B ) = P A  B  = 1  P(A  B)
= 1  [ P(A) + P(B)  P(A  B)] (Addition theorem)
= [1  P(A)]  P(B) [1  P(A)]
= P( A )P( B ).

Page no 3
Sathyabama Institute of Science and Technology
Problem 1:

From a bag containing 3 red and 2 balck balls, 2 ball are drawn at random.
Find the probability that they are of the same colour.

Solution :
Let A be the event of drawing 2 red balls
B be the event of drawing 2 black balls.
 P(A  B) = P(A) + P(B)
3C 2 2C 2 3 1 2
=  =  
5C 2 5C 2 10 10 5

Problem 2:

When 2 card are drawn from a well-shuffled pack of playing cards, what is
the probability that they are of the same suit?

Solution :
Let A be the event of drawing 2 spade cards
B be the event of drawing 2 claver cards
C be the event of drawing 2 hearts cards
D be the event of drawing 2 diamond cards.
13C 2 4
 P(A  B  C  D) = 4 = .
52C 2 17

Problem 3:

When A and B are mutually exclusive events such that P(A) = 1/2 and P(B)
= 1/3, find P(A  B) and P(A  B).

Solution :
P(A  B) = P(A) + P(B) = 5/6 ; P(A  B) = 0.

Problem 4:

If P(A) = 0.29, P(B) = 0.43, find P(A  B ), if A and B are mutually


exclusive.

Solution :
We know A  B = A
P(A  B ) = P(A) = 0.29

Problem 5:
A card is drawn from a well-shuffled pack of playing cards. What is the
probability that it is either a spade or an ace?

Solution :
Let A be the event of drawing a spade
B be the event of drawing a ace
P(A  B) = P(A) + P(B)  P(A  B)
13 4 1 4
=    .
52 52 52 13

Page no 4
Sathyabama Institute of Science and Technology
Problem 6:

If P(A) = 0.4, P(B) = 0.7 and P(A  B) = 0.3, find P( A  B ).


Solution :
P( A  B ) = 1  P(A  B)
= 1  [P(A) + P(B)  P(A  B)]
= 0.2

Problem 7:

If P(A) = 0.35, P(B) = 0.75 and P(A  B) = 0.95, find P( A  B ).


Solution :
P( A  B ) = 1  P(A  B) = 1  [P(A) + P(B)  P(A  B)] = 0.85

Problem 8:

A lot consists of 10 good articles, 4 with minor defects and 2 with major
defects. Two articles are chosen from the lot at random(with out
replacement). Find the probability that (i) both are good, (ii) both have major
defects, (iii) at least 1 is good, (iv) at most 1 is good, (v) exactly 1 is good,
(vi) neither has major defects and (vii) neither is good.

Solution :
10C2 3
(i) P( both are good) = 
16C2 8
2C2 1
(ii) P(both have major defects) = 
16C2 120
10C1 6C1  10C2 7
(iii) P(at least 1 is good) = 
16C2 8
10C0 6C2  10C1 6C1 5
(iv) P(at most 1 is good) = 
16C2 8
10C16C1 1
(v) P(exactly 1 is good) = 
16C2 2
14C2 91
(vi) P(neither has major defects) = 
16C2 120
6C2 1
(vii) P(neither is good) =  .
16C2 8

Problem 9:

If A, B and C are any 3 events such that P(A) = P(B) = P(C) = 1/4, P(A 
B) = P(B  C) = 0; P(C  A) = 1/8. Find the probability that at least 1 of the
events A, B and C occurs.

Solution :
Since P(A  B) = P(B  C) = 0; P(A  B  C) = 0
P(A  B  C) = P(A) + P(B) +P(C)  P(A  B)  P(B  C)  P(C  A) +
P(A  B  C)
3 1 5
= 00  .
4 8 8

Page no 5
Sathyabama Institute of Science and Technology
Problem 10:

A box contains 4 bad and 6 good tubes. Two are drawn out from the box at a
time. One of them is tested and found to be good. What is the probability
that the other one is also good?

Solution :
Let A be a good tube drawn and B be an other good tube drawn.
6C2 1
P(both tubes drawn are good) = P(A  B) = 
10C2 3
P( A  B ) 1/ 3 5
P(B/A) = =  (By conditional probability)
P( A ) 6 / 10 9

Problem 11:

In shooting test, the probability of hitting the target is 1/2, for a, 2/3 for B
and ¾ for C. If all of them fire at the target, find the probability that (i) none
of them hits the target and (ii) at least one of them hits the target.

Solution :
Let A, B and C be the event of hitting the target .
P(A) = 1/2, P(B) = 2/3, P(C) = 3/4
P( A ) = 1/2, P( B ) = 1/3, P( C ) = 1/4

P(none of them hits) = P( A  B  C ) = P( A )  P( B )  P( C ) = 1/24

P(at least one hits) = 1  P(none of them hits)


= 1  (1/24) = 23/24.
Problem 12:

A and B alternatively throw a pair of dice. A wins if he throws 6 before B


throws 7 and B wins if he throws 7 before A throws 6. If A begins, show that
his chance of winning is 30/61.

Solution :
Let A be the event of throwing 6
B be the event of throwing 7.

P(throwing 6 with 2 dice) = 5/36 P(throwing 7 with 2 dice) = 1/6


P(not throwing 6) = 31/36 P(not throwing 7) = 5/6

A plays in I, III, V,……trials.


A wins if he throws 6 before Be throws 7.
P(A wins) = P(A  A B A  A B A B A  …… )
= P(A) + P( A B A) + P( A B A B A) + ……
5  31 5  5  31 5  2 5
=       
36  36 6  36  36 6  36
30
=
61

Problem 13:

A and B toss a fair coin alternatively with the understanding that the first
who obtain the head wins. If A starts, what is his chance of winning?

Page no 6
Sathyabama Institute of Science and Technology
Solution :
P(getting head) = 1/2 , P(not getting head) = 1/2

A plays in I, III, V,……trials.


A wins if he gets head before B.
P(A wins) = P(A  A B A  A B A B A  …… )
= P(A) + P( A B A) + P( A B A B A) + ……
1  1 1  1  1 1 2 1
=       
2  2 2 2  2 2 2
2
=
3

Problem 14:

A problem is given to 3 students whose chances of solving it are 1/2 , 1/3


and 1/4 . What is the probability that (i) only one of them solves the problem
and (ii) the problem is solved.

Solution :
P( A solves) = 1/2 P(B) = 1/3 P(C) = 1/4
P( A ) = 1/2, P( B ) = 2/3, P( C ) = 3/4

P(none of them solves) = P( A  B  C ) = P( A )  P( B )  P( C ) =


1/4
P(at least one solves) = 1  P(none of them solves)
= 1  (1/4) = 3/4 .

Baye’s Theorem

Statement: If B1, B2, B3, ….Bn be a set of exhaustive and mutually exclusive
events associated with a random experiment and A is another event
associated with Bi, then
P ( Bi )  P ( A / Bi )
P ( Bi / A)  n
 P( Bi )  P( A / Bi )
i 1
Proof :
B1 B2 B3 B4 B5 ….. Bn
A

The shaded region represents the event A, A can occur along with B 1,
B2, B3, ….Bn that are mutually exclusive.
 AB1, AB2, AB3, …, ABn are also mutually exclusive.
Also A = AB1  AB2  AB3  … ABn
P(A) = P(AB1 ) + P(AB2 ) + P(AB3 ) + …+P(ABn)
n
=  P( ABi )
i 1
n
=  P( Bi )  P( A / Bi ) (By conditional probability)
i 1

Page no 7
Sathyabama Institute of Science and Technology
P( Bi )  P( A / Bi ) P( Bi )  P( A / Bi )
P(Bi/A) = = .
P( A ) n
 P( Bi )  P( A / Bi )
i 1

Problem 15:

Ina bolt factory machines A, B, C manufacture respectively 25%, 35%


and 40% of the total. Of their output 5%, 4% and 2% are defective bolts.
A bolt is drawn at random from the produce and is found to be defective.
What are the probabilities that it was manufactured by machines A, B
and C.

Solution :
Let B1 be bolt produced by machine A
B2 be bolt produced by machine B
B3 be bolt produced by machine C
Let A/B1 be the defective bolts drawn from machine A
A/B2 be the defective bolts drawn from machine B
A/B3 be the defective bolts drawn from machine C.
P(B1) = 0.25 P(A/B1) = 0.05
P(B2) = 0.35 P(A/B2) = 0.04
P(B3) = 0.40 P(A/B3) = 0.02

Let B1/A be defective bolts manufactured by machine A


B2/A be defective bolts manufactured by machine B
B3/A be defective bolts manufactured by machine C

3
P( A)   P( Bi )  P( A / Bi ) = (0.25)  (0.05) + (0.35)  (0.04) + (0.4) 
i 1
(0.02)
= 0.0345
P( B1 )  P( A / B1 )
P(B1/A) = = 0.3623
P( A)
P ( B2 )  P ( A / B2 )
P(B2/A) = = 0.405
P( A)
P( B3 )  P( A / B3 )
P(B3/A) = = 0.231 .
P( A)

Problem 16 :

The first bag contains 3 white balls, 2 red balls and 4 black balls. Second bag
contains 2 white, 3 red and 5 black balls and third bag contains 3 white, 4
red and 2 black balls. One bag is chosen at random and from it 3 balls are
drawn. Out of three balls two balls are white and one is red. What are the
probabilities that they were taken from first bag, second bag and third bag.

Solution :
Let P(selecting the bag) = P(Ai) = 1/3 , i = 1, 2, 3.
3
3C2 2C1 6
P(A/B1) =  P( A)   P( Bi )  P( A / Bi ) =
9C3 84 i 1
0.0746
2C2 3C1 3
P(A/B2) = 
10C3 120

Page no 8
Sathyabama Institute of Science and Technology
3C2 4C1 12
P(A/B3) = 
9C3 84
P( B1 )  P( A / B1 )
P(B1/A) = = 0.319
P( A)
P ( B2 )  P ( A / B2 )
P(B2/A) = = 0.4285
P( A)
P( B3 )  P( A / B3 )
P(B3/A) = = 0.638
P( A)

Page no 9
Sathyabama Institute of Science and Technology
Random Variable

Random Variable:
A random variable is a real valued function whose domain is the sample space of
a random experiment taking values on the real line .

Discrete Random Variable:


A discrete random variable is one which can take only finite or countable
number of values with definite probabilities associated with each one of them.

Probability mass function:


Let X be discrete random variable which assuming values x1 , x2 ,..., xn with each of
the values, we associate a number called the probability P  X  xi   p  xi  ,  i  1, 2,..., n 
this is called the probability of xi satisfying the following conditions
i. pi  0 i i.e., pi ' s are all non-negative
n
ii. p
i 1
i  p1  p2  ...  pn  1 i.e., the total probability is one.

Continuous random variable:


A continuous random variable is one which can assume every value between two
specified values with a definite probability associated with each.

Probability Density Function:


A function f is said to be the probability density function of a continuous
random variable X if it satisfies the following properties.
i. f  x   0;    x  

ii.  f  x dx  1.


Distribution Function or Cumulative Distribution Function


i. Discrete Variable:
A distribution function of a discrete random variable X is defined
as P  X  x    P  xi  .
xi  x

ii. Continuous Variable:


A distribution function of a continuous random variable X is defined
x
as F  x   P  X  x    f  x  dx .


Mathematical Expectation
The expected value of the random variable X is defined as

i. If X is discrete random variable E  X    xi p  xi  where p  x  is the probability
i 1

function of x .

Page no 10
Sathyabama Institute of Science and Technology


ii. If X is continuous random variable E  X    xf  x  dx where f  x  is the

probability density function of x .

Properties of Expectation:
1. If C is constant then E  C   C
Proof:
Let X be a discrete random variable then E  x    xp  x 
Now E  C    Cp  x 
n
 C p  x since p i  p1  p2  ...  pn  1
i 1

C
2. If a, b are constants then E  ax  b   aE  x   b
Proof:
Let X be a discrete random variable then E  x    xp  x 
Now E  ax  b     ax  b  p  x 
  axp  x    bp  x 
n
 a  xp  x   b  p  x  since p i  p1  p2  ...  pn  1
i 1

 aE  x   b
3. If a and b are constants then Var  ax  b   a 2Var  x 
Proof:
Var  ax  b   E  ax  b  E  ax  b   
2

 
 E  ax  b  aE  x   b  
2

 
 E a  x  E  x   
2 2

 
 a E  x  E  x   
2 2

 
 a Var  x  .
2

4. If a is constant then Var  ax   a 2Var  x 


Proof:
Var  ax   E  ax  E  ax   
2

 
 E  ax  aE  x   
2

 
 E a 2  x  E  x   
2

 
 a 2 E  x  E  x   
2

 
 a Var  x  .
2

Page no 11
Sathyabama Institute of Science and Technology

5. Prove that Var  x   E  x 2    E  x  


2

Proof:
Var ( x)  E  x  E  x   
2

 
 E  x 2   E  x    2 xE  x  
2

 
 E  x    2 x 
2 2

 E  x 2   E   2   E  2 x 
 E  x 2    2  2 E  x 
 E  x 2    2  2 2
 E  x2    2

Var ( x)  E  x 2    E  x  
2

Moment Generating Function (m.g.f)


A moment generating function of a random variable X (about origin) is defined
 e f  x  dx , if x is continuous
 tx

as M X  t   E e  
tX
 
 e p  x  , if x is discrete

tx

Properties of Moment Generating Function


1. M cx  t   M x  ct 
Proof:
M cx  t   E  ecxt 


 E e x ct  
 M x  ct 
2. M x  c  t   ect M x  t 
Proof:

M x  c  t   E e x  c t 
 E (e xt ect )
 ect M x  t 
3. M ax b  t   ebt M x  at 
Proof:

M ax b  t   E e ax b t 
 E  eaxt ebt 


 ebt E e x at  
 ebt M x  at 
4. If X and Y are independent random variables then M x  y  t   M x  t  .M y  t 

Page no 12
Sathyabama Institute of Science and Technology

Proof:

M x  y  t   E e  x  y t 
 E  e xt e yt 
 E  e xt  E  e yt 
M x y  t   M x  t  M y t 
Problem.1
If the probability distribution of X is given as
X : 1 2 3 4
P X : 0.4 0.3 0.2 0.1
Find P 1/ 2  X  7 / 2 X  1
Solution:
P 1/ 2  X  7 / 2   X  1
P 1/ 2  X  7 / 2 / X  1 
P  X  1
P  X  2 or 3

P  X  2,3 or 4 
P  X  2   P  X  3

P  X  2   P  X  3  P  X  4 
0.3  0.2 0.5 5
   .
0.3  0.2  0.1 0.6 6
Problem.2
A random variable X has the following probability distribution
X : 2 1 0 1 2 3
P X : 0.1 K 0.2 2 K 0.3 3K
a) Find K , b) Evaluate P  X  2  and P  2  X  2 
b) Find the cdf of X and d) Evaluate the mean of X .
Solution:
a) Since  P  X   1
0.1 K 0.2
2K 0.3 3K 1
6K 0.6 1
6K 0.4
0.4 1
K 
6 15
b) P  X  2   P  X  2, 1, 0 or 1
 P  X  2   P  X  1  P  X  0   P  X  1
1 1 1 2

  
10 15 5 15
3  2  6  4 15 1
  
30 30 2
P  2  X  2   P  X  1, 0 or 1
 P  X  1  P  X  0   P  X  1

Page no 13
Sathyabama Institute of Science and Technology

1 1 2
  
15 5 15
1 3  2 6 2
  
15 15 5
c) The distribution function of X is given by F  x  defined by
X x P( X  x) F  x   P( X  x)
-2 1 1
F  x   P( X  2) 
10 10
-1 1 1
F  x   P( X  1) 
15 6
0 2 11
F  x   P( X  0) 
10 30
1 2 1
F  x   P( X  1) 
15 2
2 3 4
F  x   P( X  2) 
10 5
3 3 F  x   P  X  3  1
15

d) Mean of X is defined by E  X    xP  x 
 1  1   1  2  3   1
E  X    2     1    0     1    2     3  
 10   15   5   15   10   5 
1 1 2 3 3 16
      .
5 15 15 5 5 15
Problem.3
A random variable X has the following probability function:
X : 0 1 2 3 4 5 6 7
2 2 2
P X : 0 K 2 K 2 K 3K K 2 K 7 K K
Find (i) K , (ii) Evaluate P  X  6  , P  X  6  and P  0  X  5 
(iii). Determine the distribution function of X .
(iv). P 1.5  X  4.5 X  2 
(v). E  3 x  4  , Var (3x  4)
1
(vi). The smallest value of n for which P  X  n   .
2
Solution:
7
(i) Since  P  X   1,
x 0

K  2K  2K  3K  K 2  2K 2  7K 2  K  1
10K 2  9K 1  0
1
K or K  1
10

Page no 14
Sathyabama Institute of Science and Technology

1
As P  X  cannot be negative K 
10
(ii) 
P X  6  
 P X  0  
 P X  1  ...  P  X  5
1 2 2 3 1 81

     ... 
10 10 10 10 100 100
Now P  X  6   1  P  X  6 
81 19
 1 
100 100
Now P  0  X  5   P  X  1  P  X  2   P  X  3  P  X  4 
 K  2K  2K  3K
8 4
 8K   .
10 5
(iii) The distribution of X is given by F  x   P  X  x 
X x P( X  x) F  x   P( X  x)
0 0 F  x   P ( X  0)  0
1 1 1
F  x   P( X  1) 
10 10
2 2 3
F  x   P( X  2) 
10 10
3 2 5
F  x   P( X  3) 
10 10
4 3 8
F  x   P( X  4) 
10 10
5 1 81
F  x   P  X  5 
100 100
6 2 83
F  x   P( X  6) 
100 100
7 17 F  x   P ( X  7)  1
100

P  x  3  P  x  4 
(iv) P 1.5  X  4.5 X  2  
1   P  x  0   P  x  1  P  x  2  
5
5
 10 
3 7
1  
10 
(v) E  x    xp ( x)
1 2 2 3 1 2 17
 1  2   3  4   5  6  7
10 10 10 10 100 100 100
E  x   3.66
E  x2    x2 p  x 

Page no 15
Sathyabama Institute of Science and Technology

1 2 2 3 1 2 17
 12   22   32   42   52   62   72 
10 10 10 10 100 100 100
E  x   16.8
2

Mean  E  x   3.66
Variance  E  x 2    E  x  
2

 16.8   3.66 
2

 3.404
1
(vi) The smallest value of n for which P  X  n   is 4
2
Problem.4
The probability mass function of random variable X is defined as P  X  0   3C 2 ,
P  X  1  4C  10C 2 , P  X  2   5C  1 , where C  0 , and P  X  r   0 if r  0,1, 2 .
Find (i). The value of C .
(ii). P  0  X  2 x  0  .
(iii). The distribution function of X .
1
(iv). The largest value of x for which F  x   .
2
Solution:
x2
(i) Since  p  x  1
x 0

p  0   p 1  p  2   1
3C 2  4C  10C 2  5C  1  1
7C 2  9C  2  0
2
C  1,
7
C  1 is not applicable
2
C 
7
The Probability distribution is
X : 0 1 2
12 16 21
P X  :
49 49 49
P  0  x  2   x  0
(ii) P 0  x  2  
 x  0 P  x  0
P  0  x  2 P  x  1
 
P  x  0 P  x  1  P  X  2
16
16
P 0  x  2   49
 x  0 16 21 37

49 49
(iii). The distribution function of X is

Page no 16
Sathyabama Institute of Science and Technology

X F  X  x  P  X  x
12
0 F  0  P  X  0   0.24
49
12 16
1 F 1  P  X  1  P  X  0   P  X  1    0.57
49 49
12 16 21
2 F  2   P  X  2   P  X  0   P  X  1  P  X  2     1
49 49 49
1
(iv) The Largest value of x for which F  x   P  X  x   is 0.
2
Problem.5
x
 ; x  1, 2,3, 4,5
If P  x   15
0 ; elsewhere
Find (i) P  X  1or 2 and (ii) P 1/ 2  X  5 / 2 x  1
Solution:
i) P  X  1 or 2   P  X  1  P  X  2 
1 2 3 1
   
15 15 15 5
 1 5 
P   X     X  1 
1  2 2
ii) P   X  / x  1   
5
2 2  P  X  1
P  X  1or 2    X  1

P  X  1
P  X  2

1  P  X  1
2 /15 2 /15 2 1
    .
1  1/15 14 /15 14 7
Problem.6
A continuous random variable X has a probability density function f  x   3x 2 ,
0  x  1 . Find ' a ' such that P  X  a   P  X  a  .
Solution:
1
Since P  X  a   P  X  a  , each must be equal to because the probability is
2
always 1.
1
 P  X  a 
2
a
1
 f  x  dx 
0
2
a
a
1  x3  1
0     a  .
2 3
3 x dx 3
2  3 0 2

Page no 17
Sathyabama Institute of Science and Technology

1
 1 3
a   
2
Problem.7
Cxe x ; if x  0
A random variable X has the p.d.f f  x  given by f  x    Find the value
0 ; if x  0
of C and cumulative density function of X .
Solution:

Since  f  x  dx  1


 Cxe
x
dx  1
0

C  x  e x    e x   1

0
C 1
 xe x ; x  0
 f  x  
 0 ;x  0
Cumulative Distribution of x is
x x
F  x    f  x dt   xe  x dx    xe  x  e  x    xe  x  e  x  1
x

0
0 0

= 1  1  x  e  x , x  0 .
Problem.8
1
  x  1 ; 1  x  1
If a random variable X has the p.d.f f  x    2 . Find the mean and
 0 ; otherwise
variance of X .
Solution:
1 1 1
Mean=1   xf  x  dx   x  x  1 dx    x 2  x  dx
1 1
1
2 1 2 1
1
1  x3 x 2  1
    
2  3 2 1 3
1
1 x x3 
1 1 4
2   x f  x dx    x  x  dx    
 2 1 3 2

1
2 1 2 4 3  1
1 1 1 1 1
     
2  4 3 4 3
1 2 1
 . 
2 3 3

 
2
Variance  2  1

Page no 18
Sathyabama Institute of Science and Technology

1 1 3 1 2
=    .
3 9 9 9
Problem.9
A continuous random variable X that can assume any value between X  2 and
X  5 has a probability density function given by f ( x)  k (1  x) . Find P  X  4  .
Solution:
k (1  x) , 2  x  5
Given X is a continuous random variable whose pdf is f  x    .
0 , Otherwise
 5
Since  f  x  dx  1   k (1  x)dx  1
 2
5
 (1  x) 2 
k  1
 2 2
 (1  5) 2 (1  2) 2 
k   1
 2 2 
 9
k 18    1
 2
 27  2
k   1  k 
2 27
 2(1  x)
 ,2  x  5
 f  x    27
0 , Otherwise
4
2
27 2
P( X  4)  (1  x) dx
4
2  (1  x)2  2  (1  4)2 (1  2) 2  2  25 9  2 16 16
          .
27  2  2 27  2 2  27  2 2  27 2 27
Problem.10
2e2 x ; x  0
A random variable X has density function given by f  x    . Find m.g.f
0 ; x  0
Solution:
 
M X t   E e tx
   e f  x  dx   e
tx tx
2e 2 x dx
0 0

 2  e t  2 x dx
0

 e t  2  x  2
 2   ,t  2.
 t  2 0 2t
Problem.11
 2 x, 0  x  b
The pdf of a random variable X is given by f  x    . For what value of b is
0, otherwise
f  x  a valid pdf? Also find the cdf of the random variable X with the above pdf.

10

Page no 19
Sathyabama Institute of Science and Technology

Solution:
 2 x, 0  x  b
Given f  x   
0, otherwise
 b
Since  f  x  dx  1   2 x dx  1
 0
b
 x2 
2   1
 2 0
b 2  0   1  b =1
 2 x, 0  x  1
 f  x  
0, otherwise
x
x
 x2 
x
F  x   P( X  x)   f  x  dx   2 xdx   2   x 2 , 0  x  1
0 0  2 0
x x
F  x   P( X  x)   f  x  dx   0 dx  0 , x  0
 
0 1 x
F  x   P( X  x)   f  x  dx   f  x  dx   f  x  dx
 0 1
1
0 1
 x2 x
  0 dx   2 x dx   0 dx =  2   1 , x  1
 0 1  2 0
 0, x0

F  x    x2 , 0  x  1
 1, x 1

Problem.12
 K
 ,  x  
A random variable X has density function f  x   1  x 2 . Determine K
 0 , Otherwise
and the distribution functions. Evaluate the probability P  x  0  .
Solution:

Since

 f  x dx  1

K

 1 x 2
dx  1

dx
K 1

1  x2
K  tan 1 x 

1


    
K     1
 2  2 
K  1
11

Page no 20
Sathyabama Institute of Science and Technology

1
K

x x
K
F  x   f  x dx   1  x dx 2
 

1  1   
  tan x     
  2 
1  
F  x    tan 1 x  ,   x  
 2 

1 1
 tan x 
dx 
P  X  0 
  1 x
1

0
2
 0

1  1  1
   tan 0   .
2  2
Problem.13
 Ke3 x , x  0
If X has the probability density function f  x   find K ,
0 , otherwise
P  0.5  X  1 and the mean of X .
Solution:

Since

 f  x  dx  1

 Ke
3 x
dx  1
0

 e3 x 
K  1
 3  0
K
1
3
K 3
1 1
 e3  e1.5 
P  0.5  X  1   f  x  dx  3  e
3 x
dx  3    e  e 
1.5 3

0.5 0.5  3 
 
Mean of X  E  x    xf  x  dx  3 xe 3 x dx
0 0

  e3 x   e3 x   3 1 1
 3x    1   
  3   9  0 9 3
1
Hence the mean of X  E  X   .
3
Problem.14
If X is a continuous random variable with pdf given by

12

Page no 21
Sathyabama Institute of Science and Technology

 Kx in 0  x  2
2 K in 2  x  4

f  x   . Find the value of K and also the cdf F  x  .
 6 K  Kx in 4  x  6
0 elsewhere
Solution:

Since  F  x  dx  1

2 4 6

 Kxdx   2Kdx    6k  kx dx  1


0 2 4

 x 2 
2 6
 x2  
6

K     2 x  2    6 x     1
4

 2 0 4
2 4 

K  2  8  4  36  18  24  8  1
8K  1
1
K
8
x
We know that F  x    f  x  dx

x
If x  0 , then F  x    f  x  dx  0

x
If x   0, 2  , then F  x    f  x  dx

0 x
F  x   f  x  dx   f  x  dx
 0
0 x 0 x
1
  0dx   Kxdx   0dx   xdx
 0 
80
x
 x2  x2
F  x     , 0  x  2
 16 0 16
0 2 x
If x   2, 4  , then F  x    f  x  dx   f  x  dx   f  x  dx
 0 2
0 2 x
  0dx   Kxdx   2Kdx
 0 2
2
 x2   x 
2 x x
x 1
  dx   dx      
0
8 2
4  16 0  4 2
1 x 1
  
4 4 2
x 4 x 1
F  x    ,2 x4
4 16 4

13

Page no 22
Sathyabama Institute of Science and Technology

0 2 4 x
If x   4, 6  , then F  x    0dx   Kxdx   2Kdx   K  6  x  dx
 0 2 4
2 4 x
x 1 1
  dx   dx    6  x  dx
0
8 2
4 4
8
2 x
 x2   x   6x x2 
4

       
 16 0  4 2  8 16 4
1 1 6 x x2
 1    3 1
4 2 8 16
4  16  8  12 x  x 2  48  16

16
 x  12 x  20
2
F  x  ,4  x  6
16
0 2 4 6 
If x  6 , then F  x    0dx   Kxdx   2Kdx   K  6  x  dx   0dx
 0 2 4 6

F  x  1 , x  6
0 ;x0
 2
x ;0  x  2
16
 1
 F  x     x  1 ;2 x4
4
 1
 16  20  12 x  x  ; 4  x  6
2


 1 ;x  6
Problem.15
2 x , 0  x  1
A random variable X has the P.d.f f  x   
0 , Otherwise
 1 1 1  3 1
Find (i) P  X   (ii) P   x   (iii) P  X  / X  
 2 4 2  4 2
Solution:
1/ 2
 1
1/ 2 1/ 2
 x2  2 1 1
(i) P  x   
 2  f  x  dx  
0 0
2 xdx  2   
 2 0 8

4
1/ 2
1 1
1/ 2
 x2 
1/ 2
(ii) P   x     f  x  dx   2 xdx  2  
4 2  1/ 4 1/ 4  2 1/ 4
1 1  1 1  3
 2        .
 8 32   4 16  16
 3 1  3
P X   X   P X  
 1
(iii) P  X  / X    
2
 
3 4 4
 4 2  1   1 
P X   P X  
 2  2

14

Page no 23
Sathyabama Institute of Science and Technology

1
 3
1 1
 x2  9 7
P  X     f  x  dx   2 xdx  2    1  
 4  3/ 4 3/ 4  2 3/ 4 16 16
1
 1
1 1
 x2  1 3
P  X     f  x  dx   2 xdx  2    1  
 2  1/ 2 1/ 2  2 1/ 2 4 4
7
 3 1 7 4 7
P  X  / X    16    .
 4 2  3 16 3 12
4
Problem.16
 1  2x
 e ,x 0
Let the random variable X have the p.d.f f  x    2 .Find the moment
0
 , otherwise.
generating function, mean & variance of X .
Solution:
 
M X  t   E  etx    etx f  x  dx   etx e  x / 2 dx
1
 0
2

  1 t  x 
1  e 2  
  1 t  x  
1   1 1
 e 2 
dx     , if t  .
20 2   1  1  2t 2
 
  2   t 
 0
d   2 
E  X    M X  t    2
2
 dt t 0  1  2t  t 0

 d2   8 
E  X 2    2 M X  t    3
8
 dt  t 0  1  2t   t 0
Var  X   E  X 2    E  X    8  4  4 .
2

Problem.17
The first four moments of a distribution about x  4 are 1,4,10 and 45 respectively. Show
that the mean is 5, variance is 3, 3  0 and 4  26 .
Solution:
Given 1  1, 2  4, 3  10, 4  45
r  r th moment about to value x  4
Here A  4
Here Mean  A  1  4  1  5

 
2
Variance  2  2  1
 4 1  3 .

 
3
3  3  321  2 1

 10  3  4 1  2 1  0
3

15

Page no 24
SCHOOL OF SCIENCE AND HUMANITIES
DEPARTMENT OF MATHEMATICS

UNIT – II - Probability Distribution – SMTA1402


Sathyabama Institute of Science and Technology

 2     
Var  X   ESMTA1402
2
- Probability and Statistics
2 
 2 2  
2
  4  Probability
 2
  2    2 Unit-2
  .
Distribution
 2  2 
Discrete type

Binomial distribution:
A random variable X is said to follow binomial distribution if it assumes only
non negative values and its probability mass function is given by
nC p x q n  x , x  0,1, 2,..., n; q  1  p
P  X  x   p( x)   x
0, otherwise
Notation: X B  n, p  read as X is following binomial distribution with parameter
n and p .
Problem.1
Find m.g.f. of Binomial distribution and find its mean and variance.
Solution:
M.G.F.of Binomial distribution:-
n
M X  t   E etx    etx P  X  x 
x 0
n
  nC x x P x q n  x etx
x 0
x

  nCx  pe q
n
t n x

x 0

M X (t )   q  pet 
n

Mean of Binomial distribution


Mean  E  X   M X   0

  n  q  pet  pet   np Since q  p  1


n 1

  t 0
E  X 2   M X   0 

 n  n  1  q  pet   pe   npet  q  pet  


n2 t 2 n 1

 t 0
E  X   n  n  1 p 2  np
2

 n 2 p 2  np 1  p   n 2 p 2  npq
Variance  E  X 2    E  X   npq
2

Mean  np ; Variance  npq


Problem.2
Comment the following: “The mean of a binomial distribution is 3 and variance is 4
Solution:
In binomial distribution, mean  variance but Variance  Mean
21

Page no 25
Sathyabama Institute of Science and Technology

Since Variance  4 &Mean  3 , the given statement is wrong.


Problem.3
 1  1
If X and Y are independent binomial variates B  5,  and B  7,  find P  X  Y  3
 2  2
Solution:
1
X  Y is also a binomial variate with parameters n1  n2  12 & p 
2
3 9
1 1 55
 P  X  Y  3  12C3      10
2 2 2
Problem.4
(i). Six dice are thrown 729 times. How many times do you expect atleast 3 dice show 5
or 6 ?
(ii) Six coins are tossed 6400 times. Using the Poisson distribution, what is the
approximate probability of getting six heads 10 times?
Solution:
(i). Let X be the number of times the dice shown 5 or 6
1 1 1
P 5 or 6   
6 6 3
1 2
 P  and q 
3 3
Here n  6
By Binomial theorem,
x 6 x
1  2
P  X  x   6Cx     where x  0,1, 2...6 .
 3  3
P  X  3  P  3  P  4   P  5   P  6 
3 3 4 2 5 6
1  2 1  2 1  2  1
 6C3      6C4      6C5      6C6  
 3  3  3  3  3  3  3
 0.3196
 Expected number of times atleast 3 dies to show 5 or 6  N  P  X  3
 729  0.3196  233 .
6
1
(ii). Probability of getting six heads in one toss of six coins is p  ,
2
6
1
  np  6400    =100
 2
e100 (100)10
Let X be the number of times getting 6 heads P( X  10)   1.025 1030
10!

Poisson distribution:
A random variable X is said to follow Poisson distribution if it assumes only non
negative values and its probability mass function is given by
 e   x
 ; x  0,1, 2,...;   0
P  X  x    x!
0, otherwise

22

Page no 26
Sathyabama Institute of Science and Technology

Notation: X P    read as X is following Poisson distribution with parameter  .


Poisson distribution as limiting form of binomial distribution:
Poisson distribution is a limiting case of Binomial distribution under the
following conditions:
(i). n the number of trials is indefinitely large, (i.e.) n  
(ii). p the constant probability of success in each trial is very small (i.e.) p  0
(iii). np   is finite.
Proof:
P  X  x   p  x   ncx p x q n  x
Let np  
 
 p , q 1 
n n
x n x
  
 p  x   n cx   1  
n   n
x n x
n!   
    1 
x ! n  x !  n   n
n n  1   n   x  1    n  x  !   x  n x

  n  1  n 
x ! n  x !    
 1  x 1 
1. 1   1  n  n x
 n x  1   
x
 
n 
 n 
x! nx 
n x
 1  x 1  x  
p  x   1. 1   1    1 
 n  n  x! n
Taking limit n   on both sides
 1   n x
x x  1   
lim p  x   lim  1  1 1  
n  x ! n   n   n  n 

x n
x  1   x  1     
 lim 1   1    lim 1   lim 1  
x ! n   n   n   n   n  n   n
 x
e 
P  X  x  ; x  0,1, 2,...
x!
Problem.1
Criticise the following statement: “The mean of a Poisson distribution is 5 while the
standard deviation is 4”.
Solution:
For a Poisson distribution mean and variance are same. Hence this statement is
not true.

23

Page no 27
Sathyabama Institute of Science and Technology

Geometric distribution:
A random variable X is said to have a Geometric distribution if it assumes only
non negative values and its probability mass function is given by
q x 1 p; x  1, 2,...;0  p  1
P  X  x  
0, otherwise
Problem.1
Find the Moment generating function of geometric distribution and find its Mean and
Variance
Solution:

M X t   E etX  

  etx q x 1 p
x 1

 pet qet 
x 1

x 1

  
2
 pet 1  qet  qet  
 

 
1
 pet 1  qet
pet
M X t  
1  qet
 d  pet   t 
1  M x (0)        pe   1
  
 dt  1  qet  
  t 0   1  qe
t

2


  t 0
p

 d2     
2  M x (0)   
pet
   
d pet   1  q
 dt 2 
   1  qe  
t
 
 t 0  

dt 1  qe t
2

  t 0
p2

1
Mean  1 
p

 
2
2
1 q  1  q
Variance  2  1  2    2
p  p p
Problem.2
State and prove Memoryless property of geometric distribution.
Solution:
If X has a geometric distribution, then for any two positive integer ' s ' and ' t '
P X  s t   P X  t .
 X  s
The p.m.f of the geometric random variable X is P( X  x)  q x 1 p , x  1, 2,3,....

 
P X  s  t  X  s P  X  s  t 
P X  s t       (1)
 X  s P  X  s P  X  s

24

Page no 28
Sathyabama Institute of Science and Technology


 P X  t  q x 1
p  qt p  qt 1 p  qt 2 p  ....  q t p 1  q  q 2  q 3  ....
x t 1

 qt p(1  q)1  qt p( p)1  qt


Hence P  X  s  t   q s t and P  X  s   q s

 
P X  s  t  X  s  q s t
(1)  P  X  s  t  s  qt  P[ X  t ]
 X  s P  X  s q
 P X  s t   P X  t
 X  s
Problem.3
1
If the probability is that a man will hit a target what is the chance that he will hit the
4
target for the first time in the 7th trial?
Solution:
The required probability is
P  FFFFFFS   P  F  P  F  P  F  P  F  P  F  P  F  P  S 
6
3 1
q p 
6
.    0.0445 .
4 4
Problem.4
A die is cast until 6 appears what is the probability that it must cast more then five
times?
Solution:
1
Probability of getting six 
6
1 1
 p  & q  1
6 6
Let x : No of throws for getting the number 6. By geometric distribution
P  X  x   q x 1 p, x  1, 2,3....
Since 6 can be got either in first, second……throws.
To find P  X  6  1  P  X  6
x 1
5 1
5
 1   .
x 1  6  6
 1   5  1   5 2  1   5 3  1   5 4  1  
 1                       
 6   6  6   6   6   6   6   6   6  
1 5 
5

1    
6   6    5 5
 1     0.4019
1
5 6
6
Problem.5
Suppose that a trainee soldier shoots a target an independent fashion. If the probability
that the target is shot on any one shot is 0.8.
(i) What is the probability that the target would be hit on 6th attempt?
(ii) What is the probability that it takes him less than 5 shots?

25

Page no 29
Sathyabama Institute of Science and Technology

Solution:
Here p  0.8, q  1  p  0.2
P  X  x   q x 1 p, x  1, 2...
(i) The probability that the target would be hit on the 6th attempt  P  X  6
  0.2   0.8  0.00026
5

(ii) The probability that it takes him less than 5 shots  P  X  5


4 4
  q x 1 p  0.8  0.2 
x 1

x 1 x 1

 0.8 1  0.2  0.04  0.008  0.9984


Continuous type
Uniform (or) Rectangular distribution:
A continuous random variable X is said to have a uniform distribution over an
interval  a, b  if its probability density function is given by
 1
 ,a  x  b
f  x  b  a
0, otherwise
Problem.1
4
If X is uniformly distributed with Mean 1 and Variance , find P  X  0
3
Solution:
If X is uniformly distributed over  a, b  , then
b  a 
2
ba
EX   and V  X  
2 12
ba
 1 a  b  2
2
b  a 
2
4
    b  a   16
2

12 3
 a  b  2 & b  a  4 We get b  3, a  1
 a  1& b  3 and probability density function of x is
1
 ; 1  x  3
f  x   4
0 ; Otherwise
0
1 1 0 1
P  x  0   dx   x 1  .
1
4 4 4
Exponential distribution:
A continuous random variable X assuming non negative values is said to have an
exponential distribution with parameter   0 , if its probability density function is
given by
 e   x , x  0
f  x  
0, otherwise
Problem.1

26

Page no 30
Sathyabama Institute of Science and Technology

Find the moment generating function of Exponential distribution and find its mean and
variance.
Solution:
 e   x , x  0
We know that f  x   
0, otherwise
 
M X  t   E  etx    etx f  x  dx    e   x etx dx
0 0

   e x  t  dx
0

 e  x   t   
  
     t  0   t
d     1
Mean  1   M X  t     2

 dt  t 0     t   t 0 

 d2     2  2
2   2 M X  t    

3
 2
 dt  t 0     t   t 0 

 
2
2 1 1
Variance  2  1  2  2  2 .
  
Problem.2
State and prove the memoryless property of exponential distribution.
Solution:
Statement:
If X is exponentially distributed with parameters  , then for any two positive
integers‘s’ and‘t’, P  x  s  t / x  s   P  x  t 
Proof:
 e   x , x  0
The p.d.f of X is f  x   
0 , Otherwise


 P  X  k     e  x dx   e   x   ek
k
k

P  x  s  t  x  s
 P  X  s  t / x  s 
P  x  s
P  X  s  t  e   s t 
    s  e  t
P  X  s e
 Px  t
Problem.3
A component has an exponential time to failure distribution with mean of 10,000 hours.
(i). The component has already been in operation for its mean life. What is the
probability that it will fail by 15,000 hours?
(ii). At 15,000 hours the component is still in operation. What is the probability
that it will operate for another 5000 hours.
Solution:

27

Page no 31
Sathyabama Institute of Science and Technology

Let X denote the time to failure of the component then X has exponential distribution
with Mean  1000 hours.
1 1
  10, 000   
 10, 000
 1 
x

 e 10,000
,x 0
The p.d.f. of X is f  x   10, 000

 0 , otherwise
(i) Probability that the component will fail by 15,000 hours given it has already been
in operation for its mean life  P  x  15, 000 / x  10, 000
P 10,000  X  15,000

P  X  10,000
15,000

 f  x  dx
e 1  e 1.5
 10,000


e 1
 f  x  dx
10,000

0.3679  0.2231
  0.3936 .
0.3679
(ii) Probability that the component will operate for another 5000 hours given that
it is in operational 15,000 hours  P  X  20, 000 / X  15, 000
 P  x  5000 [By memoryless prop]

 f  x  dx  e
0.5
  0.6065
5000

Gamma distribution:
A continuous random variable X is said to have a gamma distribution with
parameter  , if its probability density function is given by
 e  x   x 1
 ;   0, 0  x  
f  x      
0, otherwise

Problem.1
The Daily consumption of milk in a city in excess of 20,000 gallons is approximately
1
distributed as a Gamma variate with parameters   2 and   . The city has a
10, 000
daily stock of 30,000 gallons. What is the probability that the stock is in sufficient on a
particular day?
Solution:
Let X be the r.v denoting the daily consumption of milk (is gallons) in a city
Then Y  X  20,000 has Gamma distribution with p.d.f.
y
1 
f  y  y e 2 1 10,000
,y0
10, 000    2 
2

28

Page no 32
Sathyabama Institute of Science and Technology

1 6
 e [400]  25e6  .06196875
16

Normal distribution:
A random variable X is said to have a Normal distribution with parameters
 (mean) and  2 (variance) if its probability density function is given by the probability
law
1  x 
2

1   
f  x  e 2    ,   x  ,     ,   0
 2
Notation: X N   ,  2  read as X is following normal distribution with mean  and
variance  2 are called parameter.
Problem.1
t2
Prove that “For standard normal distribution N  0,1 , M X  t   e . 2

Solution:
Moment generating function of Normal distribution
 M X  t   E etx 
1  x 
2
  
1 
e
2  
 tx
e dx
 2 
x
Put z  then  dz  dx,    Z  

 z2
1 t  z    
 M X t   e 2
dz
2 
 z2 

e t  t z 

2 e

 2 
dz

1   2t 2 
t    z t 2  
e 
 e
2  2 
dz
2 
 2t 2
t  1
e e 2   z t 2

2 e

2
dz
 1
1   z t 2
the total area under normal curve is unity, we have
2 e

2
dz  1

 2t 2
t 
Hence M X  t   e 2  For standard normal variable N  0,1
2
t
M X t   e 2
Problem.2
State and prove the additive property of normal distribution.
Solution:
Statement:

30

Page no 33
Sathyabama Institute of Science and Technology

If X1 , X 2 ,..., X n are n independent normal random variates with mean  1 ,  12  ,


  ,  ,…   , 
2 2
2
n n
2
then X1  X 2  ...  X n also a normal random variable with mean
 n n

 
 i 1
 i , 
i 1
 i2  .

Proof:
We know that. M X1  X 2 ... X n  t   M X1  t  M X 2  t  ...M X n  t 
t 2 i 2
i t 
But M X i  t   e 2
, i  1, 2....n
t 12
2
t 2 22 t 2 n 2
1t   2t  n t 
M X1  X 2 ... X n  t   e 2
e 2
...e 2

 1  2 ... n t 
 1  2 ... n
2 2 2
t 2

e 2
n

n  i 2 t 2
 i t  i1 2
 e i1
By uniqueness MGF, X1  X 2  ...  X n follows normal random variable with
 n n

parameter   i ,   i 2  .
 i 1 i 1 
This proves the property.
Problem.3
X is a normal variate with mean  30 and S .D  5 Find the following P  26  X  40
Solution:
X N  30,52 
   30 &   5
X 
Let Z  be the standard normal variate

 26  30 40  30 
P  26  X  40  P  Z
 5 5 
 P  0.8  Z  2  P  0.8  Z  0  P  0  Z  2
 P  0  Z 0.8   0  z  2
 0.2881  0.4772  0.7653 .
Problem.4
The average percentage of marks of candidates in an examination is 45 will a standard
deviation of 10 the minimum for a pass is 50%.If 1000 candidates appear for the
examination, how many can be expected marks. If it is required, that double that
number should pass, what should be the average percentage of marks?
Solution:
Let X be marks of the candidates
Then X N  42,102 
X  42
Let z 
10
P  X  50  P  Z  0.8

31

Page no 34
Sathyabama Institute of Science and Technology

 0.5  P  0  z  0.8
 0.5  0.2881  0.2119
Since 1000 students write the test, nearly 212 students would pass the
examination.
If double that number should pass, then the no of passes should be 424.
We have to find z1 , such that P  Z  z1   0.424
 P  0  z  z1   0.5  0.424  0.076
From tables, z  0.19
50  x1
 z1   x1  50  10 z1
10
 50  1.9  48.1
The average mark should be 48 nearly.
Problem.5
Given that X is normally distribution with mean 10 and probability P  X  12  0.1587 .
What is the probability that X will fall in the interval  9,11 .
Solution:
Given X is normally distributed with mean   10.
x
Let z  be the standard normal variate.

12  10 2
For X  12, z  z
 
2
Put z1 

Then P  X  12  0.1587
P  Z  Z1   0.1587
 0.5  p  0  z  z1   0.1587
 P  0  z  z1   0.3413
From area table P  0  z  1  0.3413
2
 Z1  1  1

To find P 9  x  11
1 1
For X  9, z   and X  11, z 
2 2
 P 9  X  11  P  0.5  z  0.5
 2 P  0  z  0.5
 2  0.1915  0.3830
31. In a normal distribution 31% of the items are under 45 and 8% are over 64.Find the
mean and standard deviation of the distribution.
Solution:
Let  be the mean and  be the standard deviation.
Then P  X  45  0.31 and P  X  64  0.08

32

Page no 35
Sathyabama Institute of Science and Technology

45  
When X  45 , Z    z1

z1

 z1 is the value of z corresponding to the area    z dz  0.19


0

 z1  0.495
45    0.495 ---(1)
64  
When X  64 , Z   z2

z2

 z2 is the value of z corresponding to the area    z dz  0.42


0

 z2  1.405
64    1.405 ---(2)
Solving (1) & (2) We get   10 (approx) &   50 (approx)

33

Page no 36
SCHOOL OF SCIENCE AND HUMANITIES
DEPARTMENT OF MATHEMATICS

UNIT – III - Two Dimensional Random Variable – SMTA1402


Sathyabama Institute of Science and Technology
Unit.2. Two Dimensional Random Variables
SMTA1402 - Probability and Statistics

UNIT-3 TWO DIMENSIONAL


QUESTION RANDOM VARIABLE
BANK ANWERS

1. Let X and Y have joint density function f  x, y   2, 0  x  y  1 .Find the marginal density
function. Find the conditional density function Y given X  x .
Solution:
Marginal density function of X is given by

fX  x  f  x   f  x, y  dy

1 1
  f  x, y  dy   2dy  2  y  x
1

x x

 2 1  x  , 0  x  1.
Marginal density function of Y is given by

fY  y   f  y    f  x, y  dx

y

  2dx  2 y, 0  y  1 .
0

Conditional distribution function of Y given X  x is f y 


x
f  x, y 
f  x
  2

1
2 1  x  1  x
.

0 , x  a

1  x 
2. Verify that the following is a distribution function. F  x      1 , a  x  a .
2  a 
1 ,x  a
Solution:
F  x  is a distribution function only if f  x  is a density function.
d 1
f  x   F  x    , a  x  a
dx 2a

 f  x  1

a
1 1 1
 dx   x  a   a   a 
a

a
2a 2a 2a
1
.2a  1 .
2a
Therefore, it is a distribution function.

Page no 37
Sathyabama Institute of Science and Technology
Unit.2. Two Dimensional Random Variables

x2

3. Prove that  f  x  dx  p  x
x1
X 1  x  x2 

Solution:
x2

 f  x  dx   F  x 
x2
X X x1
x1

 FX  x2   FX  x1 
 P  X  x2   P  X  x1 
 P  x1  X  x2 

4. A continuous random variable X has a probability density function f  x   3x 2 , 0  x  1 .


Find ' a ' such that P  X  a   P  X  a  .
Solution:
1
Since P  X  a   P  X  a  , each must be equal to because the probability is always 1.
2
1
 P  X  a 
2
a
1
 f  x  dx 
0
2
a
a
1  x3  1
0     a  .
2 3
3 x dx 3
2  3 0 2
1
 1 3
a   
2

 Ae x  y , 0  x  y, 0  y  
5. Suppose that the joint density function f  x, y    Determine A .
0 , otherwise
Solution:
Since f  x, y  is a joint density function
 

  f  x, y  dxdy  1 .
 
 y
   Ae x e y dxdy  1
0 0
 y
 e x 
 A e   dy  1
y

0  1 0

 A e  y  e 2 y  dy  1
0

Page no 38
Sathyabama Institute of Science and Technology
Unit.2. Two Dimensional Random Variables


 e  y e 2 y 
 A   1
  1 2 0
1
 A  1 A  2
2
6. Examine whether the variables X and Y are independent, whose joint density function is
f  x, y   xe  x y 1 , 0  x, y   .
Solution:
The marginal probability function of X is
 
fX  x  f  x   f  x, y  dy   xe  x y 1 dy
 0

 e x y 1 
   0  e   e ,
x x
 x
  x 0
The marginal probability function of Y is
 
fY  y   f  y    f  x, y  dx   x e
 x  y 1
dx
 0

  e   e  x y 1  

 x  y 1
 x    2 
    y  1  0   y  1  0
1

 y  1
2

1
Here f  x  . f  y   e x   f  x, y 
1  y 
2

 X and Y are not independent.

7. If X has an exponential distribution with parameter 1. Find the pdf of y  x


Solution:
Since y  x , x  y 2
Since X has an exponential distribution with parameter 1, the pdf of X is given by
f X  x   e x , x  0  f  x   e x ,   1
dx
 fY  y   f X  x 
dy
 e x 2 y  2 ye y
2

fY  y   2 ye y , y  0
2

  
8. If X is uniformly distributed random variable in   ,  , Find the probability density
 2 2
function of Y  tanX .
3

Page no 39
Sathyabama Institute of Science and Technology
Unit.2. Two Dimensional Random Variables

Solution:
Given Y  tanX  x  tan 1 y
dx 1
 
dy 1  y 2
  
Since X is uniformly distribution in   ,  ,
 2 2
1 1
fX  x  
ba  
2 2
1  
fX  x  ,   x 
 2 2
dx 1  1 
Now fY  y   f X  x    ,    y  
dy   1  y 2 
1
 fY  y   ,   y  
 1  y 2 

9. If the Joint probability density function of  x, y  is given by f  x, y   24 y 1  x  ,


0  y  x  1 Find E  XY  .
Solution:
1 1
E  xy     xyf  x, y  dxdy y
0 y
1 1
 24  xy 2 1  x  dxdy x y
0 y

 1 y 2 y3 
1
4
 24 y     dy  .
2
x
0 6 2 3 15

10. If X and Y are random Variables, Prove that Cov  X , Y   E  XY   E  X  E Y 


Solution:
cov  X , Y   E  X  E  X   Y  E Y  
 E  XY  XY  YX  XY 
 E  XY   XE Y   YE  X   XY
 E  XY   XY  XY  XY
 E  XY   E  X  E Y   E  X   X , E Y   Y 

11. If X and Y are independent random variables prove that cov  x, y   0


Proof:

Page no 40
Sathyabama Institute of Science and Technology
Unit.2. Two Dimensional Random Variables

cov  x, y   E  xy   E  x  E  y 
But if X and Y are independent then E  xy   E  x  E  y 
cov  x, y   E  x  E  y   E  x  E  y 
cov  x, y   0.

12. Write any two properties of regression coefficients.


Solution:
1. Correction coefficients is the geometric mean of regression coefficients
2. If one of the regression coefficients is greater than unity then the other should be less
than 1.
y 
bxy  r and byx  r x
x y
If bxy  1 then byx  1 .

13. Write the angle between the regression lines.


The slopes of the regression lines are
y 1y
m1  r , m2 
x r x
If  is the angle between the lines, Then
 x y 1  r 2 
tan   2
 x   y2  r 

When r  0 , that is when there is no correlation between x and y, tan    (or)  
2
and so the regression lines are perpendicular
When r  1 or r  1 , that is when there is a perfect correlation ve or ve ,   0 and so the
lines coincide.

14. State central limit theorem


Solution:
If X 1 , X 2 .... X n is a sequence of independent random variable E  X i   i and
Var  X i    i2 , i  1, 2,....n and if Sn  X 1  X 2  ......  X n then under several conditions S n
n n
follows a normal distribution with mean    i and variance  2    i2 as n   .
i 1 i 1

15. i). Two random variables are said to be orthogonal if correction is zero.
ii). If X  Y then correlation coefficients between them is 1.

Part-B

Page no 41
Sathyabama Institute of Science and Technology
Unit.2. Two Dimensional Random Variables

16.a). The joint probability density function of a bivariate random variable X ,Y is


k  x  y  , 0  x  2, 0  y  2

f XY  x, y    where ' k ' is a constant.

 0 , otherwise
i. Find k .
ii. Find the marginal density function of X and Y .
iii. Are X and Y independent?
iv. Find fY y
X
  x Y
 
and f X x .
y
Solution:
(i). Given the joint probability density function of a brivate random variable  X , Y  is
 K  x  y  , 0  x  2, 0  y  2

f XY  x, y   

0 , otherwise
   
Here  f XY  x, y  dxdy  1    K  x  y  dxdy  1
   
2
2 2
 x2
2

0 0 K  x  y  dxdy  1  K 0  2  xy  dy  1
0
2
 K   2  2 y  dy  1
0
2
 K  2 y  y 2   1
0

 K 8  0   1
1
K
8
(ii). The marginal p.d.f of X is given by
 2
1
f X  x    f  x, y  dy    x  y  dy

80
2
1 y2  1 x
  xy   
8 2 0 4
 The marginal p.d.f of X is
 x 1
 , 0 x2
fX  x   4
0 , otherwise
The marginal p.d.f of Y is
 2
1
fY  y    f  x, y  dx    x  y  dx

80
2
1  x2 
   yx 
8 2 0

Page no 42
Sathyabama Institute of Science and Technology
Unit.2. Two Dimensional Random Variables

1 y 1
 2  2 y 
8 4
The marginal p.d.f of Y is
 y 1
 , 0 y2
fY  y    4
0 , otherwise
(iii). To check whether X and Y are independent or not.
f X  x  fY  y  
 x  1  y  1  f x, y
XY  
4 4
Hence X and Y are not independent.

X
 
(iv). Conditional p.d.f fY y is given by
x
1
 x  y 1  x  y
fY y 
X
  x
f  x ,
fX  x
y  8
1

 x  1 2  x  1
4

X
 
fY y  
x
1 x y
2  x 1 
 , 0  x  2, 0  y  2
 1  2
(v) P 
0  y  2


x  1  Y X x  1
 0
 f y
 dy 
 
1

1 2 1 y 5
  dy  .
20 2 32

17.a). If X and Y are two random variables having joint probability density function
1
  6  x  y  , 0  x  2, 2  y  4
f  x, y    8 Find (i) P  X  1  Y  3
0 , otherwise


(ii) P  X  Y  3 (iii) P X  1Y  3 . 
b). Three balls are drawn at random without replacement from a box containing 2 white, 3 red
and 4 black balls. If X denotes the number of white balls drawn and Y denotes the number of
red balls drawn find the joint probability distribution of X , Y .
Solution:
a).
y 3 x 1
P  X  1  Y  3    f  x, y  dxdy
y  x 
y 3 x 1
1
   8  6  x  y  dxdy
y  2 x 0

Page no 43
Sathyabama Institute of Science and Technology
Unit.2. Two Dimensional Random Variables

3 1
1
 6  x  y  dxdy
8 2 0

1
1  
3
x2
  6 x   xy  dy
82 2 0
3
1 11  1 11y y 2 
3
    y  dy    
822  8 2 2 2
3
P  X  1  Y  3 
8
1 3 x
1
(ii). P  X  Y  3     6  x  y  dydx
0 2
8
3 x
1  y2 
1
  6 y  xy   dx
80 2 2

1   3  x 
1 2

  6  3  x   x  3  x    12  2 x  2 dx
8 0  2 

1 
1
  18  6 x  3x  x 
 2 
9  x2  6 x  
 10  2 x   dx
80 2 

1  
1
9 x2 6 x
  18  9 x  x 2     10  2 x dx
80 2 2 2 
1 7 x2 
1

8 0  2
   4 x   dx
2
1
1  7 x 4 x 2 x3  1  7 1
       2 
8 2 2 6 0 8  2 6
1  21  12  1 1  10  5
     .
8 6  8  6  24
P  x  1  y  3

(iii). P X  1
Y 3
 
P  y  3
2
The Marginal density function of Y is fY  y    f  x, y dx
0
2
1
  6  x  y dx
0
8
2
1 x2 
 6 x   yx 
8 2 0
1
 12  2  2 y 
8
8

Page no 44
Sathyabama Institute of Science and Technology
Unit.2. Two Dimensional Random Variables

5 y
 , 2  y  4.
4
x 1 y 3
1
  8  6  x  y  dxdy

P X 1
Y 3
  x 0 y  2
y 3

 fY  y  dy
y 2

3 3
3 8  8
 5 y 
3
1 y2 
2  4  dy
4 
5y  
2 2
3 8 3
   .
8 5 5

b). Let X takes 0, 1, 2 and Y takes 0, 1, 2 and 3.


P  X  0, Y  0   P( drawing 3 balls none of which is white or red)
P ( all the 3 balls drawn are black)
4C 4  3  2 1 1
 3  .
9C3 98 7 21
P  X  0, Y  1  P( drawing 1 red ball and 2 black balls)
3C1  4C2 3
= 
9C3 14
P  X  0, Y  2   P( drawing 2 red balls and 1 black ball)
3C2  4C1 3  2  4  3 1
   .
9C3 98 7 7
P  X  0, Y  3  P( all the three balls drawn are red and no white ball)
3C3 1
= 
9C3 84
P  X  1, Y  0   P( drawing 1White and no red ball)
2 43
2C  4C2
= 1  1 2
9C3 98 7
1 2  3
12  1 2  3 1
  .
98 7 7
P  X  1, Y  1  P( drawing 1White and 1 red ball)
23
2C1  3C1 9  8  7 2
  
9C3 1 2  3 7

Page no 45
Sathyabama Institute of Science and Technology
Unit.2. Two Dimensional Random Variables

P  X  1, Y  2   P( drawing 1White and 2 red ball)


2C1  3C2 2  3  2 1
  
9C3 9  8  7 14
1 2  3
P  X  1, Y  3  0 (Since only three balls are drawn)
P  X  2, Y  0   P( drawing 2 white balls and no red balls)
2C2  4C1 1
 
9C3 21
P  X  2, Y  1  P( drawing 2 white balls and no red balls)
2C2  3C1 1
 
9C3 28
P  X  2, Y  2   0
P  X  2, Y  3  0
The joint probability distribution of X , Y may be represented as
Y
0 1 2 3
X
1 3 1 1
0
21 14 7 84
1 2 1
1 0
7 7 14
1 1
2 0 0
21 28

18.a). Two fair dice are tossed simultaneously. Let X denotes the number on the first die and Y
denotes the number on the second die. Find the following probabilities.
 
(i) P  X  Y   8 , (ii) P  X  Y  8 , (iii) P  X  Y  and (iv) P X  Y  6 Y  4 .
b) The joint probability mass function of a bivariate discrete random variable  X , Y  in given by
the table.
X
Y 1 2 3
1 0.1 0.1 0.2
2 0.2 0.3 0.1
Find
i. The marginal probability mass function of X and Y .
ii. The conditional distribution of X given Y  1 .
iii. P  X  Y  4 
Solution:
a). Two fair dice are thrown simultaneously

10

Page no 46
Sathyabama Institute of Science and Technology
Unit.2. Two Dimensional Random Variables

1,11, 2  ... 1, 6  


 
 2,1 2, 2  ...  2, 6  
S   , n( S )  36
 . . ... . 
 6,1 6, 2  ...  6, 6  
 
Let X denotes the number on the first die and Y denotes the number on the second die.
1
Joint probability density function of  X , Y  is P  X  x, Y  y   for
36
x  1, 2,3, 4,5, 6 and y  1, 2,3, 4,5, 6
(i) X  Y   the events that the no is equal to 8 
  2, 6  ,  3,5 ,  4, 4  ,  5,3 ,  6, 2 
P  X  Y  8   P  X  2, Y  6   P  X  3, Y  5   P  X  4, Y  4 
 P  X  5, Y  3  P  X  6, Y  2 
1 1 1 1 1 5
     
36 36 36 36 36 36
(ii) P  X  Y  8
 2, 6  
 
 3,5  ,  3, 6  
 
X  Y   4, 4  ,  4,5  ,  4, 6  
 
 5,3 ,  5, 4  5,5  ,  5, 6  
 6, 2  ,  6,3 ,  6, 4  ,  6,5  6, 6  
 
 P  X  Y  8  P  X  Y  8   P  X  Y  9   P  X  Y  10 
 P  X  Y  11  P  X  Y  12 
5 4 3 2 1 15 5
      
36 36 36 36 36 36 12
(iii) P  X  Y 
P  X  Y   P  X  1, Y  1  P  X  2, Y  2   ......  P  X  6, Y  6 
1 1 1 6 1

  ..........   
36 36 36 36 6
P  X  Y  6  Y  4

(iv) P X  Y  6
Y 4
 
P Y  4 
1
Now P  X  Y  6  Y  4  
36
6
P Y  4  
36

11

Page no 47
Sathyabama Institute of Science and Technology
Unit.2. Two Dimensional Random Variables

1

P X Y  6
Y 4  1
 36  .
6 6
36
b). The joint probability mass function of  X , Y  is
X
1 2 3 Total
Y
1 0.1 0.1 0.2 0.4
2 0.2 0.3 0.1 0.6
Total 0.3 0.4 0.3 1
From the definition of marginal probability function
PX  xi    PXY  xi , y j 
yj

When X  1 ,
PX  xi   PXY 1,1  PXY 1, 2 
 0.1  0.2  0.3
When X  2 ,
PX  x  2   PXY  2,1  PXY  2, 2 
 0.2  0.3  0.4
When X  3 ,
PX  x  3  PXY  3,1  PXY  3, 2 
 0.2  0.1  0.3
 The marginal probability mass function of X is
0.3 when x  1

PX  x   0.4 when x  2
0.3 when x  3

The marginal probability mass function of Y is given by PY  y j    PXY  xi , y j 
xi
3
When Y 1 , PY  y  1   PXY  xi ,1
xi 1

 PXY 1,1  PXY  2,1  PXY  3,1


 0.1  0.1  0.2  0.4
3
When Y 2 , PY  y  2    PXY  xi , 2 
xi 1

 PXY 1, 2   PXY  2, 2   PXY  3, 2 


 0.2  0.3  0.1  0.6
 Marginal probability mass function of Y is
0.4 when y  1
PY  y   
0.6 when y  2
(ii) The conditional distribution of X given Y 1 is given by

12

Page no 48
Sathyabama Institute of Science and Technology
Unit.2. Two Dimensional Random Variables


P X x
Y 1   P  XPYx  1Y  1
From the probability mass function of Y , P y 1 Py 1 0.4
P  X  1  Y  1
When X 
1, P X 1
Y 1
 
P Y  1
PXY 1,1 0.1
   0.25
PY 1 0.4
P  2,1 0.1
When X 
2, P X  2
Y 1 
P 1

XY

Y0.4
 0.25

P  3,1 0.2
When X 
3 , P X  3 Y  1  XY
PY 1

0.4
 0.5

(iii). P  X  Y  4   P  x, y  / x  y  4 Where x  1, 2,3; y  1, 2


 P 1,1 , 1, 2  ,  2,1
 PXY 1,1  PXY 1, 2   PXY  2,1
 0.1  0.1  0.2  0.4
19.a). If X and Y are two random variables having the joint density function
1
f  x, y    x  2 y  where x and y can assume only integer values 0, 1 and 2, find the
27
conditional distribution of Y for X x .
b). The joint probability density function of X ,Y is given by
 2 x2
 xy  , 0  x  2, 0  y  1
f XY  x, y    8 . Find (i) P X 1 , (ii) P X Y and
0
 , otherwise
(iii) P X Y 1
Solution:
a). Given X and Y are two random variables having the joint density function
1
f  x, y    x  2 y     (1)
27
Where x 0,1, 2 and y 0,1, 2
Then the joint probability distribution X and Y becomes as follows

Y
0 1 2 f1  x 
X
1 2 3
0 0
27 27 27

13

Page no 49
Sathyabama Institute of Science and Technology
Unit.2. Two Dimensional Random Variables

2 3 4 9
1
27 27 27 27
4 5 6 15
2
27 27 27 27
The marginal probability distribution of X is given by f1  X    P  x, y  and is calculated in
j

the above column of above table.


The conditional distribution of Y for X is given by f1 Y  y  X x  
f  x, y 
f  x
1
and is obtained in

the following table.


X
0 1 2
Y
1 2
0 0
3 3
1 3 5
1
9 9 9
1 1 1
2
6 3 2
P  X  0, Y  0 

P Y 0
X 0  
P  X  0

0
6
0

27
2
     27  1
P Y 1
X 0

P
 X 0, Y
P  X  0
1
6 3
27
4
     27  2
P Y 2
X 0

P
X 0, Y
P  X  0
2
6 3
27
1
P  X  1, Y  0  27 1
P Y 0
X 1
  P  X  1

9 9

27
3
P  X  1, Y  1 27 3 1
P Y 1
X 1
  P  X  1
  
9 9 3
27
5
     27  5
P Y 2
X 1

P
 X 1, Y
P  X  1
2
9 9
27

14

Page no 50
Sathyabama Institute of Science and Technology
Unit.2. Two Dimensional Random Variables

2
P  X  2, Y  0  27 1

P Y 0
X 2
 
P  X  2

12 6

27
4
P  X  2, Y  1 27 1

P Y 1
X 2
 
P  X  2

12 3

27
6
P  X  2, Y  2  27 1

P Y 2
X 2
 
P  X  2

12 2

27
x2
b). Given the joint probability density function of X ,Y is f XY  x  y   xy 2  ,
8
0  x  2, 0  y  1

(i). P  X  1   f X  x  dx
1
1
The Marginal density function of X is f X  x    f  x, y  dy
0

 1
x  2
f X  x     xy 2   dy y y 1
0
8 
1
 xy 2 x 2 y  x x2
    , 1 x  2
 3 8  0 3 8
2
 x x2 
P  X  1      dx x
1
3 8 
2
 x 2 x3  19
    . y x 1 x 2
 6 24 1 24
(ii) P  X  Y     f XY  x, y  dxdy y 1
R2

 2 x2 
1 y

P  X  Y      xy   dxdy y x
y 0 x 0 
8 
y
1
 x 2 y 2 x3 
    dy x 0 x
0 
2 24  0
1
1
 y 4 y3   y5 y 4 
     dy    
0
2 24   10 96  0
1 1 96  10 53
   
10 96 960 480

15

Page no 51
Sathyabama Institute of Science and Technology
Unit.2. Two Dimensional Random Variables

(iii) P  X  Y  1   f XY  x, y  dxdy
R3

Where R3 is the region


1 1 y
 2 x2 
P  X  Y  1  y0 x0  xy  8  dxdy y

1 y
1
 x 2 y 2 x 3  
      dy y 1
y  0 
2 24   0
 1  y 2 y 2 1  y 3 
1
   dy x y 1

y 0 
2 24 

 

1  1 y2  2 y y2
 
1 y 
3

dy x
0
 2 24 

1
  y 3 y 5 2 y 2  1 1  y 4 
       x 1
  3 5 4 2 96 
0
1 1 1 1 13
     .
6 10 4 96 480

20).a). If the joint distribution functions of X and Y is given by


1  e 1  e  , x  0, y  0
x y

F  x, y   
0 , otherwise
i. Find the marginal density of X and Y .
ii. Are X and Y independent.
iii. P 1 X 3, 1 Y 2 .
b). The joint probability distribution of X and Y is given by
6  x  y
 , 0  x  2, 2  y  4
f  x, y    8 . Find P 1 Y 3 X 2 .
0 , otherwise
Solution:
 
a). Given F  x, y   1  e x 1  e y 
 1  e x  e y  e x y 
The joint probability density function is given by
 2 F  x, y 
f  x, y  
xy
 
2
  1  e x  e y  e x  y  
xy

16

Page no 52
Sathyabama Institute of Science and Technology
Unit.2. Two Dimensional Random Variables

   y  x  y  

e e
x  
 x  y 

e , x  0, y  0
 f  x, y   

0 , otherwise
(ii) The marginal probability function of X is given by
f  x  f X  x
 
f  x, y  dy   e
 x  y 
 
 0
dy

 e  x  y  
 
 1  0

  e   
 x y
 0
x
e , x0
The marginal probability function of Y is
f  y   fY  y 

  f  x, y  dx



  e  x  y  dx   e  x  y  
0
0

 e y , y  0
 f  x  f  y   e xe y  e  f  x, y 
 x  y 

 X and Y are independent.


(iii) P 1  X  3,1  Y  2   P 1  X  3  P 1  Y  2  [Since X and Y are independent]
3 2
  f  x dx   f  y  dy.
1 1
3 2
  e x dx   e y dy
1 1
3 2
 e   e y 
x
    
 1 1  1 1

 e3  e1 e2  e1  
 e 5  e 4  e 3  e 2
3

b). P 1 Y 3 f y dy
X 2 x 2
1
4
f X  x    f  x, y  dy
2

17

Page no 53
Sathyabama Institute of Science and Technology
Unit.2. Two Dimensional Random Variables

 6 x y 
4
  dy
2 
8
4
1 y2 
  6 y  xy  
8 2 2
1
 16  4 x  10  2 x 
8
6 x y

 
f y 
x
f    8  6 x y
x ,
f  x
y
6  2x 6  2x
8
3

P1 Y 3 f y dy
X 2 x 2
1

 4 y 
3
  dy
2
2 
3
1 y2 
 4 y  
2 2 2
3
1 y2  1 17  11
  4 y    14    .
2 2 2 2  2 4
SMT1205 - Probability and Statistics
21).a). Two random variables X and Y have the following joint probability density
2  x  y, 0  x  1, 0  y  1
function f  x, y    . Find the marginal probability density function
0 , otherwise
of X and Y . Also find the covariance between X and Y .
6 x y
b). If f  x, y   , 0  x  2, 2  y  4 for a bivariate X , Y , find the correlation
8
coefficient
Solution:
2  x  y, 0  x  1, 0  y  1
a) Given the joint probability density function f  x, y   
0 , otherwise

Marginal density function of X is f X  x    f  x, y  dy

1
   2  x  y  dy
0
1
 y2 
  2 y  xy  
 2 0
1
 2 x
2

18

Page no 54
SCHOOL OF SCIENCE AND HUMANITIES
DEPARTMENT OF MATHEMATICS

UNIT – IV - Correlation and Regression – SMTA1402


Sathyabama Institute of Science and Technology
Unit.2. Two Dimensional Random Variables

 6 x y 
4
  dy
2 
8
4
1 y2 
  6 y  xy  
8 2 2
1
 16  4 x  10  2 x 
8
6 x y

 
f y 
x
f    8  6 x y
x ,
f  x
y
6  2x 6  2x
8
3

P1 Y 3 f y dy
X 2 x 2
1

 4 y 
3
  dy
2
2 
3
1 y2 
SMTA1402 - Probability and Statistics
 2
3
1 y2  1 17  11
 Unit-4
 4 y    14   and
Correlation . Regression
2 2 2 2 4

21).a). Two random variables X and Y have the following joint probability density
2  x  y, 0  x  1, 0  y  1
function f  x, y    . Find the marginal probability density function
0 , otherwise
of X and Y . Also find the covariance between X and Y .
6 x y
b). If f  x, y   , 0  x  2, 2  y  4 for a bivariate X , Y , find the correlation
8
coefficient
Solution:
2  x  y, 0  x  1, 0  y  1
a) Given the joint probability density function f  x, y   
0 , otherwise

Marginal density function of X is f X  x    f  x, y  dy

1
   2  x  y  dy
0
1
 y2 
  2 y  xy  
 2 0
1
 2 x
2

18

Page no 55
Sathyabama Institute of Science and Technology
Unit.2. Two Dimensional Random Variables

3
  x, 0  x  1
fX  x   2
0 , otherwise
1
Marginal density function of Y is fY  y     2  x  y  dx
0
1
 x2 
  2 x   xy 
 2 0
3
 y
2
3
  y, 0  y  1
fY  y    2
0 , otherwise
Covariance of  X , Y   Cov  X , Y   E  XY   E  X  E Y 
1
1
3
1
  3 x 2 x3  5
E  X    xf X  x  dx   x   x  dx     
0    2 2 3  0 12
0
2
3 
1 1
5
E Y    yfY  y  dy   y   y  dy 
0 0 2  12
Cov  X , Y   E  XY   E  X  E Y 
1 1
E  XY     xy f  x, y  dxdy
0 0
1 1
   xy  2  x  y  dxdy
0 0
1 1
    2xy  x 2 y  xy 2  dxdy
0 0
1
1
 2 x 2 y x3 x2 
   y  y 2  dy
0 
2 3 2 0
1
 1 y2 
   y    dy
0
3 2 
1
 y 2 y y3  1
    
 2 3 6 0 6
1 5 5
Cov  X , Y    
6 12 12
1 25 1
   .
6 144 144

19

Page no 56
Sathyabama Institute of Science and Technology
Unit.2. Two Dimensional Random Variables

E  XY   E  X  E Y 
b). Correlation coefficient  XY 
 XY
Marginal density function of X is

 6 x y  6  2x
4
fX  x  f  x, y  dy  2  8  dy  8
Marginal density function of Y is

 6 x y  10  2 y
2
fY  y    f  x, y  dx     dx 
0 

8 8
 6  2x 
2 2
Then E  X    xf X  x  dx   x   dx
0 
0
8 
2
1  6 x 2 2 x3 
   
8 2 3 0
1 16  1 20 5
 12     
8 13  8 3 6
4
 10  2 y  1 10 y 2 2 y 3 
4
17
E Y    y   dy      6
2  8  8 2 3 2
2
 6  2x  1  6 x3 2 x 4 
2 2
E X  2
  x f x  x  dx   x 
2

 8 
 dx 2

8 3

4 0
 1
0 0
4
 10  2 y  1 10 y 3 2 y 4 
4
E Y    y 
25
2 2
 dy     
2  8  8 3 4 2 3
2
 5  11
 
Var  X    X2  E X 2   E  X   1    
2

 6  36
2
25  17  11
Var Y     E Y     E Y  
2
2
Y
2
  
3  6  36
 6 x y 
4 2
E  XY     xy   dxdy
2 0  8 
2
1  6 x 2 y x3 y x 2 y 2 
4
     dy
82 2 3 2 0
4
1  2 1 12 y 2 8 y 2 2 y 3 
4
8
  12 y  y  2 y dy     
8 2 3  8 2 3 2 3 2
1 64 128 16 16  1  56 
 96    24      
8 3 3 3 3  8 3 
7
E  XY  
3

20

Page no 57
Sathyabama Institute of Science and Technology
Unit.2. Two Dimensional Random Variables

7  5  17 
   
E  XY   E  X  E Y  3  6  6 
 XY  
 XY 11 11
6 6
1
 XY   .
11
1
22.a). Let the random variables X and Y have pdf f  x, y   ,  x, y    0, 0  , 1,1 ,  2, 0  .
3
Compute the correlation coefficient.
b) Let X1 and X 2 be two independent random variables with means 5 and 10 and standard
devotions 2 and 3 respectively. Obtain the correlation coefficient of UV where U  3 X1  4 X 2
and V  3 X1  X 2 .
Solution:
a). The probability distribution is

X 0 1 2 P Y 
Y
1 1
0 3 0 0 3
1 1
1 0 3 0 3
1 1
0 0 0 3 3
P X  1 1 1
3 3 3

 1  1  1
E  X    xi pi  xi    0    1    2    1
i  3  3  3
 1  1  1 1
E Y    yi p j  y j    0    1    0   
j  3  3  3 3
 1  1  1 5
E  X 2    xi p  xi    0    1    4   
2

i  3  3  3 3
Var ( X )  E  X 2    E  X     1 
2 5 2
3 3
 1  1  1 1
E  Y 2    y j p  y j    0     1    0   
2

j  3  3  3 3
V Y   E Y 2    E Y     
2 1 1 2
3 9 9

21

Page no 58
Sathyabama Institute of Science and Technology
Unit.2. Two Dimensional Random Variables

E  XY   E  X  E Y 
Correlation coefficient  XY 
V  X  V Y 
E  XY    xi y j p  xi , y j 
i j

1 1 1 1
 0.0.  0.1.0  1.0.0  1.1.  1.2.0  0.0.0  0.1.0  0.2. 
3 3 3 3
1 1
 1  
 XY 
3 3  0
2 2

3 9
Correlation coefficient  0 .
b). Given E  X1   5, E  X 2   10
V  X1   4, V  X 2   9
Since X and Y are independent E  XY   E  X  E Y 
E UV   E U  E V 
Correlation coefficient 
Var U Var V 
E U   E  3 X1  4 X 2   3E  X1   4E  X 2 
  3  5   4 10   15  40  55.
E V   E  3 X1  X 2   3E  X1   E  X 2 
  3  5  10  15  10  5
E UV   E  3 X1  4 X 2  3 X1  X 2 
 E 9 X1  3 X1 X 2  12 X1 X 2  4 X 2 
2 2

   
 9E X1  3E  X1 X 2   12E  X1 X 2   4E X 2
2 2

 9E  X   9E  X X   4E  X 
2 2
1 1 2 2

 9E  X   9E  X  E  X   4E  X 
2 2
1 1 2 2

 9E  X   450  4E  X 
2 2
1 2

V  X   E  X    E  X 
2 2
1 1 1

E  X   V  X    E  X   4  25  29
2 2
1 1 1

E  X   V  X    E  X   9  100  109


2 2
2 2 2

 E UV    9  29   450   4 109 


 261  450  436  275
Cov(U ,V )  E UV   E U  E V 

22

Page no 59
Sathyabama Institute of Science and Technology
Unit.2. Two Dimensional Random Variables

 275   5  55  0
Since Cov U ,V   0, Correlation coefficient  0 .
1 1
23.a). Let the random variable X has the marginal density function f  x   1,   x  and
2 2
 1
1, x  y  x  1,   x  0
 

 2
let the conditional density of Y be f y   . Prove that the
x 1
1,  x  y  1  x, 0  x 
 2
variables X and Y are uncorrelated.
b). Given f  x, y   xe   , x  0, y  0 . Find the regression curve of Y on X .
 x y 1

Solution:
1 1 1
2
 x2  2 2
a). We have E  X    xf  x  dx   xdx     0

1

1  2 1
2 2 2
1
0 x 1 2 1 x
E  XY     xydxdy    xydxdy

1 x 0 x
2
1
0
 x 1  2
1 x 
  x   ydy dx   x   ydy dx
1  x  0  x 

2
1
0 2
1 1

2  x  2 x  1 dx  2  x 1  2 x  dx
1 0

2
1
0
1  2 x3 x 2  1  x 2 2 x3  2
        0
2 3 2 1 2  2 3 0
2

Since Cov  X , Y   E  XY   E  X  E Y   0 , the variables X and Y are uncorrelated.

b). Regression curve of Y on X is E y  x


 x    yf  y x  dy

E y


f  x, y 
f  y / x 
fX  X 

Marginal density function f X  x    f  x, y  dy
0

23

Page no 60
Sathyabama Institute of Science and Technology
Unit.2. Two Dimensional Random Variables


 x  y 1
 x e dy
0

 e  x y 1 
 x   e x , x  0
  x 0

Conditional pdf of Y on X is f y 
x  
f  x, y  xe xy  x
f  x
  x  xe xy
e X

The regression curve of Y on X is given by

 x

E y   yxe xy dy
0

 e xy e xy 
 xy  2 
 x x 0

  1 1
E y   y  and hence xy  1 .
x x x
x y
 , 0  x  1, 0  y  2
24.a). Given f  x, y    3 , obtain the regression of Y on X and X on
0 , otherwise
Y.
b). Distinguish between correlation and regression Analysis
Solution:
a). Regression of Y on X is E Y X  
 X   yf  y x  dy

E Y 


 X   ff  x,xy
f Y
X

fX  x   f  x, y  dy

2
 x y 1 y2 
2
   dy   xy  
0
3  3 2 0
2  x  1

3
f  x, y 
 
f Y
X
 
x y
f X  x  2( x  1)
y  x  y
 X
2
Regression of Y on X E Y  dy
0
2  x  1

24

Page no 61
Sathyabama Institute of Science and Technology
Unit.2. Two Dimensional Random Variables

2
1  xy 2 y 3 
  
2  x  1  2 3 0
1  8 3x  4
  2x   
2  x  1  3  3  x  1

 Y    xf  x y  dx

E X


 y   ff x,yy
f x
Y

fY  y    f  x, y  dx

1
 x y 1  x2 
1
   dx    xy 
0
3  3 2 0
1 1 
   y
3 2 

 
f x 
y
2 x  y
2 y 1

 Y x y
1
Regression of X on Y  E X  dx
0
2 y  1
1
1  x2 
   xy 
2 y 1  2 0
1
y
2 1
  .
2 y 1 2

b).
1. Correlation means relationship between two variables and Regression is a Mathematical
Measure of expressing the average relationship between the two variables.
2. Correlation need not imply cause and effect relationship between the variables. Regression
analysis clearly indicates the cause and effect relationship between Variables.
3. Correlation coefficient is symmetric i.e. rxy  ryx where regression coefficient is not symmetric
4. Correlation coefficient is the measure of the direction and degree of linear relationship
between two variables. In regression using the relationship between two variables we can predict
the dependent variable value for any given independent variable value.

25.a). X any Y are two random variables with variances  x2 and  y2 respectively and r is the
y x
coefficient of correlation between them. If U  X  KY and V  X  , find the value of k
y
so that U and V are uncorrelated.

25

Page no 62
Sathyabama Institute of Science and Technology
Unit.2. Two Dimensional Random Variables

b). Find the regression lines:


X 6 8 10 18 20 23
Y 40 36 20 14 10 2
Solution:
Given U  X  KY
E U   E  X   KE Y 
X
VX Y
Y

E V   E  X   X E Y 
Y
If U and V are uncorrelated, Cov U ,V   0
E U  E U   V  E V    0
    
 E  X  KY  E  X   KE Y     X  X Y  E  X   X E Y     0
  Y Y 

   
 E   X  E  X   K Y  E Y      X  E  X    X Y  E Y      0

  Y 
 X  2
 E  X  E  X     X  E  X  Y  E Y    K Y  E Y    X  E  X    K X Y  E Y     0
2

 Y Y 
X 
V  X   Cov  X , Y   KCov  X , Y   K X V Y   0
Y Y
   
K Cov  X , Y   X V Y    V  X   X Cov  x, y 
 Y  Y
X
V  X   r 
Y X Y  X2  r X2
K 
 r X  Y   X  Y
r X  Y  X V Y 
Y
 X 1  r 
2

  X .
 X  Y 1  r  Y

b).
X Y X2 Y2 XY
6 40 36 1600 240

26

Page no 63
Sathyabama Institute of Science and Technology
Unit.2. Two Dimensional Random Variables

8 36 64 1296 288
10 20 100 400 200
18 14 324 196 252
20 10 400 100 200
23 2 529 4 46
X 85 Y 122 X2 1453 Y2 3596 XY 1226

 x 85  y 122
X   14.17 , Y    20.33
n 6 n 6
 x2   x 
2 2
1453  85 
x         6.44
n  n  6  6
 y2   y 
2 2
3596  122 
y        13.63
n  n  6  6 
 xy 1226
xy  14.17  20.33
r n  6  0.95
 x y  6.44 13.63
 6.44
bxy  r x  0.95   0.45
y 13.63
y 13.63
byx  r  0.95   2.01
x 6.44
The regression line X on Y is
 
x  x  bxy y  y  x  14.17  0.45 y  y  
 x  0.45 y  23.32
The regression line Y on X is
 
y  y  byx x  x  y  20.33  2.01 x  14.17 
 y  2.01x  48.81

26. a) Using the given information given below compute x , y and r . Also compute  y when
 x  2, 2 x  3 y  8 and 4 x  y  10 .
b) The joint pdf of X and Y is
X
Y -1 1

27

Page no 64
Sathyabama Institute of Science and Technology
Unit.2. Two Dimensional Random Variables

1 3
0 8 8
2 2
1 8 8
Find the correlation coefficient of X and Y .
Solution:
a). When the regression equation are Known the arithmetic means are computed by solving the
equation.
2 x  3 y  8 ------------ (1)
4 x  y  10 ------------ (2)
(1)  2  4 x  6 y  16 ------- (3)
 2   3  5 y  6
6
y
5
6
Equation 1  2 x  3    8
5
18
 2x  8 
5
11
x
5
11 6
i.e. x  & y 
5 5
To find r , Let 2 x  3 y  8 be the regression equation of X on Y .
3
2x  8  3y  x  4  y
2
3
 bxy  Coefficient of Y in the equation of X on Y  
2
Let 4 x  y  10 be the regression equation of Y on X
 y  10  4 x
 byx  coefficient of X in the equation of Y on X  4 .
r   bxybyx
 3
      4 
 2
 bxy & byx are negative 
 2.45
Since r is not in the range of  1  r  1 the assumption is wrong.
Now let equation 1 be the equation of Y on X
8 2x
 y 
3 3
 byx  Coefficient of X in the equation of Y on X

28

Page no 65
Sathyabama Institute of Science and Technology
Unit.2. Two Dimensional Random Variables

2
byx  
3
from equation (2) be the equation of X on Y
1
bxy  
4
2 1
r   bxy byx      0.4081
3 4
2
To compute  y from equation  4  byx  
3
y
But we know that byx  r
x
2 
  0.4081 y
3 2
  y  3.26
b). Marginal probability mass function of X is
1 3 4
When X  0, P  X    
8 8 8
2 2 4
X  1, P  X    
8 8 8
Marginal probability mass function of Y is
1 2 3
When Y  1, P Y    
8 8 8
3 2 5
Y  1, P Y    
8 8 8
4 4 4
E  X    x p  x   0   1 
x 8 8 8
3 5 3 5 2
E Y    y p  y   1  1    
y 8 8 8 8 8

E  X 2    x 2 p  x   02   12  
4 4 4
x 8 8 8

E Y 2    y p  y    1   12     1
2 2 3 5 3 5
y 8 8 8 8
V  X   E  X 2    E  X 
2

2
4 4 1
   
8 8 4
V  Y   E Y 2    E  Y  
2

2
 1  15
 1   
 4  16

29

Page no 66
Sathyabama Institute of Science and Technology
Unit.2. Two Dimensional Random Variables

E  XY    xy p  x, y 
x y

1 3 2 2
 0   0    1  1    0
8 8 8 8
1 1 1
Cov  X , Y   E  XY   E  X  E Y   0    
2 4 8
1
Cov  X , Y  
r  8  0.26 .
V  X  V Y  1 15
4 16

27. a) Calculate the correlation coefficient for the following heights (in inches) of fathers X and
their sons Y .
X 65 66 67 67 68 69 70 72
Y 67 68 65 68 72 72 69 71
b) If X and Y are independent exponential variates with parameters 1, find the pdf of
U  X Y .
Solution:

X Y XY X2 Y2
65 67 4355 4225 4489
66 68 4488 4359 4624
67 65 4355 4489 4285
68 72 4896 4624 5184
69 72 4968 4761 5184
70 69 4830 4900 4761
72 71 5112 5184 5041
X 544 Y 552 XY 37560 X2 37028 Y2 38132

 x 544
X   68
n 8
 y 552
Y   69
n 8
XY  68  69  4692
1 2 1
X   x2  X  (37028)  682  4628.5  4624  2.121
n 8
1 1
Y   y2  y2   38132   692  4766.5  4761  2.345
n 8

30

Page no 67
Sathyabama Institute of Science and Technology
Unit.2. Two Dimensional Random Variables

1 1
Cov  X , Y    XY  X Y   37650   68  69
n 8
 4695  4692  3
The correlation coefficient of X and Y is given by
Cov  X , Y  3
r  X ,Y   
 XY  2.121 2.345
3
  0.6032 .
4.973
b). Given that X and Y are exponential variates with parameters 1
f X  x   e x , x  0, fY  y   e y , y  0
Also f XY  x, y   f X  x  f y  y  since X and Y are independent
 e  xe  y
 e  x y  ; x  0, y  0
Consider the transformations u  x  y and v  y
 x u v, y  v v
x x
  x, y  u v 1  1
J   1
  u, v  y y 0 1
u v
fUV  u , v   f XY  x, y  J  e  x e  y  e u  v e  v
 eu 2v , u  v  0, v  0 RI R II
In Region I when u  0 v u
 
f u    f  u, v  dv   e
u
.e 2v dv u
u u

 e2 v 
u
e  
 2  u
eu eu
 0  e 2u  
2 2
In Region II when u  0

f  u    f (u, v)dv
0

eu
  eu  2v  dv 
0
2
 eu
 2 , u  0
 f  u    u
e , u  0
 2

31

Page no 68
Sathyabama Institute of Science and Technology
Unit.2. Two Dimensional Random Variables

28. a) The joint pdf of X and Y is given by f  x, y   e   , x  0, y  0 . Find the pdf of


 x y

X Y
U .
2
b) If X and Y are independent random variables each following N  0, 2  , find the pdf of
Z  2 X  3Y . If X and Y are independent rectangular variates on  0,1 find the distribution of
X
.
Y
Solution:
x y
a). Consider the transformation u  &v  y
2
 x  2u  v and y  v
x x
  x, y  u v 2  1
J   2
  u, v  y y 0 1
u v
fUV  u, v   f XY  x, y  J
 e x y  2  2e x y  2e 2u vv
 2e2u , 2u  v  0, v  0
u
fUV  u , v   2e 2u , u  0, 0  v 
2
u u
2 2
f  u    fUV  u, v  dv   2e 2u dv
0 0
u
  2e2u v  2
0

 u 2u
2 e , u  0
f u    2
0 , otherwise

b).(i) Consider the transformations w  y ,


i.e. z  2 x  3 y and w  y
1
i.e. x   z  3w  , y  w
2
x x
1 3
  x, y  z w  1
J    2 2  .
  z, w  y y 2
0 1
z w
Given that X and Y are independent random variables following N  0, 2 

32

Page no 69
Sathyabama Institute of Science and Technology
Unit.2. Two Dimensional Random Variables


 x2  y 2 
1
 f XY  x, y   e 8 ,   x, y  
8
The joint pdf of  z, w is given by
f ZW  z, w  J f XY  x, y 
1 
   z  3 w   w2 
2
4 
1 1
 . e 8
2 8
1  321  z 3w2  4 w2 
 e ,   z, w   .
16
The pdf of z is the marginal pdf obtained by interchanging f ZW  z, w w.r.to w over the range of
w.
1

  321  z 2 6 wz 13w2  
 fZ  z     e dw
16  
z 2     w2 
13  6 wz  3 z   3 z   
2 2
     
e  e 
1  32 32  13      
13 13
 dw
16  
 
 
   13  w 3 z  
2 2 2
z 9z
1  32   
  
 dw
1332 32  13 
e e
16 
 
2
13
1  8z13  t2

16
e 

e 32
dt

13 2 13 16 r 32
r t  dr  tdt  dr  dt  dr  dt
32 16 13t 13
1
16 13 4 
dr  dt  r 2 dr  dt
13 r 32 13  2
z 1 2

2 4  
 e 813  e r r 2 dr
16 13  2 0
z 1 2

1  
 e 813  e r r 2 dr
2 13  2 0
z2
z2 
2 2 13 
2
1  1
 e 813
  e
2 13  2 2 13 2
i.e. Z 
N 0, 2 13 
b).(ii) Given that X and Y are uniform Variants over  0,1
1, 0  x  1 1, 0  y  1
 f X  x   and fY  y   
0, otherwise 0, otherwise

33

Page no 70
Sathyabama Institute of Science and Technology
Unit.2. Two Dimensional Random Variables

Since X and Y are independent,


1, 0  x, y  1
f XY  x, y   f X  x  f y  y  
0, otherwise
x
Consider the transformation u  and v  y
y
i.e. x  uv and y  v
x x
  x, y  u v v 0
J   v
  u, v  y y u 1
u v
 fUV  u, v   f XY  x, y  J
 v, 0  u  , 0  v  
The range for u and v are identified as follows.
0  x  1 and 0  y  1.
 0  uv  1 and 0  v  1
 uv  0, uv  1, v  0 and v  1
 uv  0 and v  0  u  0
Now f  u    fUV  u, v  dv
The range for v differs in two regions
1
f  u    fUV  u, v dv
0
1
1
 v2  1
  vdv     , 0  u  1
0  2 0 2
1
u
f  u    fUV  u, v dv
0
1
1 u
u
 v2  1
  vdv     2 , 1  u  
0  2  0 2u
1
 2 , 0  u  1
 f u   
 1 , u 1
 2u 2

29. a) If X 1 , X 2 ,..... X n are Poisson variates with parameter   2 . Use the central limit theorem
to estimate P 120  Sn  160  where sn  X 1  X 2  ......  X n and n  75 .

34

Page no 71
SCHOOL OF SCIENCE AND HUMANITIES
DEPARTMENT OF MATHEMATICS

UNIT – V - Analysis of Variance and Statistical Quality Control –


SMTA1402
Sathyabama Institute of Science and Technology
SMTA1402 - Probability and Statistics
Unit - V Analysis of Variance and Statistical Quality Control
ANOVA (Analysis of Variance) :
Analysis of Variance is a technique that will enable us to test for the significance of the
difference among more than two sample means.
Assumptions of analysis of variance:
(i) The sample observations are independent
(ii) The environmental effects are additive in nature
(iii) The samples have been randomly selected from the population.
(iv) Parent population from which observations are taken in normal.
One Way Classification (or) Completely randomized Design (C.R.D)
The C.R.D is the simplest of all the designs, based on principles of randomization and
replication. In this design, treatments are allocated at random to the experimental units over the
entire experimental materials.
Advantages of completely randomized block design:
The advantages of completely randomized experimental design as follows:
(i) Easy to lay out. (ii) Allow flexibility (iii) Simple statistical analysis
(iv) lots of information due to missing data is smaller than with any other design
Two Way Classification (or) Randomized Block Design (R.B.D):
The entire experiment influences on only two factors is two way Classification.
The basic principles of design of experiments:
(i) Randomization (ii) Replication (iii) Local Control
Working Procedure ( One – Way classification )

Null Hypothesis H 0 : There is no significance difference between the treatments.

Alternate Hypothesis H 1 : There is a significance difference between the treatments.


Analysis:
Step 1: Find N= number of observations
Setp 2: FindT = The total value of observations

T2
Step 3: Find the correction Factor = C.F 
N
Step 4: Calculate the total sum of squares = TSS    X12   X 22   X 32  ...  C.F

Step 4: Find Total Sum of Square TSS    X12   X 22   X 32  ...  C.F

Page no 72
Sathyabama Institute of Science and Technology
  X 2  X 2  X 2 
Step 5: Column Sum of Square SSC    1

 2

 3
 ...   C.F
 N1 N2 N3 
 
Where N i  Total number of observation in each column ( i  1, 2,3,... )
Step 6: Prepare the ANOVA TABLE to calculate F-ratio.

Source of Sum of Degree


Mean Square F- Ratio
Variation Degrees of freedom
MSC
Between SSC FC  if MSC  MSE
SSC c-1 MSC  MSE
Columns c 1
(or)
MSE
SSE FC  if MSE  MSC
Error SSE N-c MSE  MSC
Nc

Total

Step 7: Find the table value (use  2 table)


Step 8: Conclusion:
Calculated value < Table Value, the we accept Null Hypothesis H 0 (or)
Calculated value > Table Value, the we rejectNull Hypothesis H 0
Working Procedure ( Two – Way classification )

Null Hypothesis H 0 : There is no significance difference between the treatments.

Alternate Hypothesis H 1 : There is a significance difference between the treatments.

Analysis:
Step 1: Find N= number of observations
Setp 2: FindT = The total value of observations

T2
Step 3: Find the correction Factor = C.F 
N
Step 4: Calculate the total sum of squares = TSS    X12   X 22   X 32  ...  C.F

Step 4: Find Total Sum of Square TSS    X12   X 22   X 32  ...  C.F


  X 2  X 2  X 2
Step 5: Find column sum of Square SSC  
 1   2   3  ...  C.F
 N1 N2 N3 
 
Where N i  Total number of observation in each column (i  1, 2,3,...)

Page no 73
Sathyabama Institute of Science and Technology
  Y 2  Y 2  Y 2 
Step 6: Find Row sum of square = SSR    1

 2

 3
 ...   C.F
 N1 N2 N3 
 
Where N j  Total number of observation in each Row ( j  1, 2,3,...)
Step 7:Prepare the ANOVA TABLE to calculate F-ratio.

Source of Sum of Degree


Mean Square F- Ratio
Variation Degrees of freedom
MSC
FC  if MSC  MSE
MSE
Between SSC
SSC c-1 MSC  (or)
Columns c 1
MSE
FC  if MSE  MSC
MSC
MSR
FR  if MSR  MSE
MSE
Between SSR
SSR r-1 MSC  (or)
Rows r 1
MSE
FR  if MSE  MSR
MSR
SSE
Error SSE N-c-r+1 MSE 
N  c  r 1

Total TSS rc-1

Step 8: Find the table value for both FC & FR (use  2 table)
Step 9:Conclusion:
Calculated value < Table Value, the we accept Null Hypothesis H 0 (or)
Calculated value > Table Value, the we reject Null Hypothesis H 0

1 The following are the numbers of mistakes made in 5 successive days of 4 technicians
. working for a photographic laboratory :

Tech I (X 1 ) Tech II (X 2 Tech III (X 3 ) Tech IV(X 4


) )
6 14 10 9
14 9 12 12
10 12 7 8
8 10 15 10
11 14 11 11

Test at the level of significance  = 0.01 whether the differences among the 4 samples
means can be attributed to chance.
Solution:
H0: There is no significant difference between the technicians

Page no 74
Sathyabama Institute of Science and Technology
H1 : Significant difference between the technicians
We shift the origin
X1 X2 X3 X4 TOTAL X12 X22 X32 X42
-4 4 0 -1 -1 16 16 0 1
4 -1 2 2 7 16 1 4 4
0 2 -3 -2 -3 0 4 9 4
-2 0 5 0 3 4 0 25 0
1 4 1 1 7 1 16 1 1
Total -1 9 5 0 13 37 37 39 10

N= Total No of Observations = 20 T=Grand Total = 13

(Grand total )2
Correction Factor = = 8.45
Total No of Observations

TSS   X12  X22  X32  X 42  C.F  37  37  39  10  8.45  114.55

( X1 )2 ( X 2 )2 ( X3 )2 (1)2 (9)2 (5)2


SSC     C.F     0  8.45  12.95
N1 N1 N1 5 5 5
SSE = TSS – SSC = 114.55-12.95= 101.6
ANOVA Table
Source of Sum of Degree of
Mean Square F- Ratio
Variation Squares freedom

Between SSC
SSC=12.95 C-1= 4-1=3 MSC  =4.317
Samples K 1 MSC
FC 
MSE
Within SSE =1.471
SSE=101.6 N-C=20-4=16 MSE  =6.35
Samples NK

Cal FC = 1.471 & Tab FC (16,3) = 5.29


Conclusion : Cal FC < Tab FC  There is no significance difference between the
technicians
2 A completely randomized design exprement with 10 plots and 3 treatments gave the
. following results.
Plot No 1 2 3 4 5 6 7 8 9 10
Treatment A B C A C C A B A B
Yield 5 4 3 7 5 1 3 4 1 7
Analyse the results for treatment effects.
Solution:

Page no 75
Sathyabama Institute of Science and Technology
A B C
5 4 3
7 4 5
3 7 1
1
Null Hypothesis H0: There is no significant difference in treatments
Alternate Hypothesis H1 : Significant difference in treatments
X1 X2 X3 TOTAL X12 X22 X32
5 4 3 12 25 16 9
7 4 5 16 49 16 25
Total
3 7 1 11 9 49 1
1 1 1
16 15 9 40 84 81 35
Step1: N= Total No of Observations = 10
Step 2: T=Grand Total = 40
(Grand total )2T 2 402
Step 3: Correction Factor =  =  160
N 10
Total No of Observations
Step 4: TSS   X1   X 2   X 3  C.F  84  81  35 160  40
2 2 2

 X    X    X 
2 2 2
(16) 2 152
SSC   C.F    3  160
1 2 3
Step 5:
N1 N1 N1 4 3
SSC  64  75  27  160  6
Where N 1 = Number of elements in each column
Step 7: SSE=TSS-SSC  40  6  34
Step 8: ANOVA TABLE:
Source of Sum of Degree of
Mean Square F- Ratio
Variation Squares freedom

SSC
MSC 
Between C 1
SSC=6 C-1= 3-1=2 MSE
Columns 6 FC 
 3 MSC
2
4.86
SSE 
MSE  3
NC
Error SSE=34 N-C=10-3=7  1.62
34
  4.86
87

Cal FC = 1.62

Page no 76
Sathyabama Institute of Science and Technology
Table value : FC (7,2)=19.35
Conclusion : Cal FC< Tab FC
We accept Null Hypothesis  There is no significance difference in tretments
3 The following table gives the number of articles of a product produced by five
. different workers using four types of machines.

Machines
Workers
P Q R S
A 44 38 47 36
B 46 40 52 43
C 34 36 44 32
D 43 38 46 33
E 38 42 49 39

Test (i) Whether the five workers differ with respect to mean productivity and
(ii) Whether the four machines differ with respect to mean productivity.
Solution: H0: There is no significant difference between the Machine types and no
significant difference between the Workers
H1 :Significant difference between the Machine types and no significant difference
between the Workers

We shift the origin Xij = xij – 46; h = 5; k = 4; N = 20

A B C D Total=Ti* [Ti*2]/k X*ij2


1 -2 -8 1 -10 -19 90.25 169
2 0 -6 6 -3 -3 2.25 81
3 -12 -10 -2 -14 -38 361 444
4 -3 -8 0 -13 -24 144 242
5 -8 -4 3 -7 -16 64 138
Total=T*j -25 -36 8 -47 -100 661.5 1074
[T*j2]/h 125 259.2 12.8 441.8 838.8

T=Grand Total = -100


(Grand total ) 2 (100) 2
Correction Factor =   500
Total No of Observatio ns 20
TSS   X ij2  C.F  1074  500  574
i j

T
2

SSR   C.F  661.5  500  161.5


i*

Page no 77
Sathyabama Institute of Science and Technology
T
2

SSC   C.F  838.8  500  338.8


*j

h
SSE = TSS – SSC – SSR = 574 – 161.5 – 338.8 = 73.7
ANOVA Table

Source of Sum of Degree of Mean


F- Ratio FTabRatio
Variation Squares freedom Square
Between
MSR=
Rows SSR=161.5 h - 1= 4 FR = 6.574 F5%(4, 12) =
40.375
(Workers) 3.26
Between
MSC =
Columns SSC=338.8 k – 1=3
112.933
(Machine) FC =
(h – 1)( k – MSE 18.388
F5%(3, 12) =
Residual SSE = 73.7 3.59
1) = 12 = 6.1417
Total 1074

Conclusion : Cal FC< Tab FC and Cal FR< Tab FR  There is no significant difference
between the Machine types and no significant difference between the Workers

4 A Company appointments four salesmen A, B, C and D and observes their sales in 3


. seasons: summer, winter and monsoon. The figures (in lakhs of Rs.) are given in the
following table:
Salesman
Season
A B C D
Summer 36 36 21 35
Winter 28 29 31 32
Monsoon 26 28 29 29
i) Do the salesmen significantly differ in performance?
ii) Is there significant difference between the seasons?
Solution:
Null Hypothesis H 0 : There is no significant difference between the sales in the 3 seasons and
also between the sales of the 4 salesmen.
Alternate Hypothesis H 1 : There is a significant difference between the sales in the 3 seasons
and also between the sales of the 4 salesmen.
Test statistic:
To simplify calculations we deduct 30 from each value

Page no 78
Sathyabama Institute of Science and Technology
Seasons A B C D Seasons
X1 2 X22 X32 X42
X1 X2 X3 X4 Total

Y1 Summer 6 6 -9 5 8 36 36 81 25

Y2 Winter -2 -1 1 2 0 4 1 1 4

Y3 Monson -4 -2 -1 -1 -8 16 4 1 1

Total 0 3 -9 6 0 56 41 83 30
Step1: N= Total No of Observations = 12
Step 2: T=Grand Total = 0
T 2 02
(Grand total )2
Step 3: Correction Factor =  0 =
N 12
Total No of Observations
Step 4: TSS   X1   X 2   X 3   X 4  C.F  56  41 83  30  0  210
2 2 2 2

Step 5:
 X    X    X    X 
2 2 2 2
02 32 (9)2 62
SSC   C.F     0
1 2 3 4

N1 N1 N1 N1 3 3 3 3
SSC  0  3  27  12  0  42
Where N 1 = Number of elements in each column
Step 6:
Y   Y   Y 
2 2 2
82 02 (8)2 62
SSR   C.F      0  16  0  16  0  32
1 2 3

N2 N2 N2 4 4 4 4
Where N 2 = Number of elements in each row
Step 7: SSE=TSS-SSC-SSR  210  42  32
Step 8: ANOVA TABLE:
Source of Sum of Degrees of Mean Sum of varience F – ratio
Variation Squares Freedom Squares
Between SSC=42 c-1=4-1=3 SSC MSE
Columns MSC  MSC 
c 1 MSC
(Salesmen) 42 22.67 FC (6,3)  8.94
  14 
3 14
 1.619
Between SSR =32 r-1=3-1=2
MSR 
SSR
MSR 
MSE FR (6,2)  8.94
rows r 1 MSR
(Seasons) 32 22.67
  16 
2 16
 1.417

Page no 79
Sathyabama Institute of Science and Technology
Error SSE=136 N-c-r +1=6 SSE
MSE 
N  c  r 1
136
  22.67
6
Total 210 11
Table Value of F = FC (Error,d.f)  FC (6,3)  8.94 , FR (Error,d.f)  8.94 with 5% level of
significance
Conclusion:
1) Cal FR < Table FR ,0.05 (6,3)
Hence we accept the H 0 and we conclude that there is no significant difference between sales
in the three seasons.
2) Cal FR < Table FR ,0.05 (6, 2) .
Hence we accept the H 0 and we conclude that there is no significant difference between in
the sales of 4 salesmen.
5 Analyze 22 factorial experiments for the following table.
.
Replications
Treatment
I II III IV
(1) 12 12.3 11.8 11.6
a 12.8 12.6 13.7 14
b 11.5 11.9 12.6 11.8
ab 14.2 14.5 14.4 15
SOLUTION:
Null hypothesis: All the mean effects are equal.
Let A and B be the two factors.
Let n=number of replications=4
Subtract 12 from each

Replications
Treatment
I II III IV
(1) 0 0.3 -0.2 -0.4
a 0.8 0.6 1.7 2
b -0.5 -0.1 0.6 -0.2
ab 2.2 2.5 2.4 3

Page no 80
Sathyabama Institute of Science and Technology

Let us find SS for the table


2
Replications Row Ri

Treatment I II III IV Total


Ri

(1) 0 0.3 -0.2 -0.4 -0.3 0.09


a 0.8 0.6 1.7 2 5.1 26.01
b -0.5 -0.1 0.6 -0.2 -0.2 0.04
ab 2.2 2.5 2.4 3 10.1 102.01
Column 2.5 3.3 4.5 4.4 T=14.7
Total C j

Cj
2 6.25 10.89 20.25 19.36

T=14.7
2
Correction factor= T =13.5
N

TSS=21.19
SSC=0.688
SSR=18.54
SSE=1.962

Source of Sum of Degree of Mean FTab


F- Ratio
Variation Squares freedom Square Ratio
FB  7.409 10.56
b S B =1.63 1 MSB=1.63

FA  70.04
10.56
a S A =15.41 1 MSA=15.41

10

Page no 81
Sathyabama Institute of Science and Technology
FAB  6.81
10.56
ab S AB =1.50 1 MSAB=1.50

Error SSE=1.962 N-C-r+1=9 SSE=1.962

Cal( F A )=70.04  H 0 is rejected at 1% level

Cal( FB )=7.409  H 0 is accepted at 1% level

Cal( FAB )=10.56  H 0 is accepted at 1% level


6 Analyse the variance in the following latin square of yields (in kgs) of paddy where A,
. B, C, D denote the different methods of cultivation.

D 122 A 121 C 123 B 122


B 124 C 123 A 122 D 125
A 120 B 119 D 120 C 121
C 122 D 123 B 121 A 122
Examine whether the different methods of cultivation have given significantly
different yields.

Solution:

We shift the origin Xij = xij – 100; n = 4; N = 16

I II III IV Total=Ti* [Ti*2]/n X*ij2


A 2 1 3 2 8 16 18
B 4 3 2 5 14 49 54
C 0 -1 0 1 0 0 2
D 2 3 1 2 8 16 18
Total=T*j 8 6 6 10 30 81 92
[T*j2]/n 16 9 9 25 59
Xi*2 24 20 14 34 92

11

Page no 82
Sathyabama Institute of Science and Technology

Letters Total=Ti* [Ti*2]/n


P 1 2 0 2 5 6.25
Q 2 4 -1 1 6 9
R 3 3 1 2 9 20.25
S 2 5 0 3 10 25
Total 30 60.5
(Grand total ) 2 (30) 2
T=Grand Total = 30 ;Correction Factor = 
Total No of Observatio ns 16
(30) 2
TSS   X  C.F  92  2
ij  35.75
i j 16

T
2
(30) 2
SSR   C.F  81   24.75
i*

n 16
 T* j
2
(30) 2
SSC   C.F  59   2.75
n 16
T
2
(30) 2
SSL   C.F  60.5   4.25
i*

n – SSC – SSR-SSL16= 35.75 – 24.75 – 2.75 – 4.25 = 4


SSE = TSS

ANOVA Table
Source of Sum of Degree of Mean FTabRatio
F- Ratio
Variation Squares freedom Square ( 5% level)

Between
SSR=24.75 n - 1= 3 MSR=8.25
Rows FR=
FR(3,
12.31
Between
SSC=2.75 n - 1= 3 MSC = 0.92
Columns 6)=4.76
Between FC = 1.37
SSL = 4.25 n - 1= 3 MSL = 1.42
Letters
Fc(3, 6)=4
FL = 2.12
(n – 1)(n – 2) .76
Residual SSE= 4 MSE = 0.67
=6

12

Page no 83
Sathyabama Institute of Science and Technology
FL(3, 6)=4

.76

Total 35.75

Conclusion :
Cal FC< Tab FC , Cal FL< Tab FL and Cal FR> Tab FR  There is significant difference
between the rows , no significant difference between the letters and no significant
difference between the columns

7 A variable trial was conducted on wheat with 4 varieties in a Latin Square Design.
. The plan of the experiment and the per plot yield are given below :

C 25 B 23 A 20 D 20
A 19 D 19 C 21 B 18
B 19 A 14 D 17 C 20
D 17 C 20 B 21 A 15
Analyse data and interpret the result.
H0 : Four varieties are similar
H1 : Four varieties are not similar
Let us take 20 as origin for simplifying the calculation

Variety X1 X2 X3 X4 TOTAL X12 X22 X32 X42

Y1 5 3 0 0 8 25 9 0 0

Y2 -1 -1 1 -2 -3 1 1 1 4

Y3 -1 -6 -3 0 -10 1 36 9 0

Y4 -3 0 1 -5 -7 1 0 1 25
0 -4 -1 -7 -12 9 46 11 29

N=Total No of Observations = 16 T=Grand Total = -12

(Grand total )2
Correction Factor = =9
Total No of Observations

13

Page no 84
Sathyabama Institute of Science and Technology
TSS   X  X  X  X
1
2
2
2
3
2
4
2
 C.F  36  46  11  29  9  113

SSC 
( X ) 1
2

( X 2)
2

( X 3)
2
 C.F 
(6) 2 (10 ) 2 (6) 2 (10 ) 2
   9  4
N1 N1 N1 4 4 4 4

SSR 
( Y ) 1
2

( Y ) 2
2

( Y ) 3
2

( Y ) 4
2
 C.F 
(8) 2 (3) 2 (10 ) 2 (7) 2
    9  46 .5
N1 N2 N2 N2 4 4 4 4

To find SSK
Treatment 1 2 3 4 Total
A 0 -1 -6 -5 -12
B 3 -2 -1 1 1
C 5 1 0 0 6
D 0 -1 -3 -3 -7
( Y ) 1
2

( Y ) 2
2

( Y ) 3
2

( Y ) 4
2
 C.F
SSK= K1 K2 K3 K4
 48.5
SSE= TSS  SSCSSRSSK = 113-7.5-46.5-48.5=10.5
ANOVA Table
Source of Sum of Degree of
Mean Square F- Ratio
Variation Squares freedom
SSC
Column MSC  MSC
SSC=7.5 n-1=3 n 1 FC  =1.43
Treatment MSE
=2.5
SSR
Row MSR  MSE
SSR=46.5 n-1=3 n 1 FR  =8.86
Treatments MSR
=15.5
SSK
Between MSK  MSK
SST=48.5 n-1=3 n 1 FK  =9.24
Treatments MSE
=16.17
MSE
Error (or) (n-1) (n- SSE
SSE=10.5 
Residual 2)=6 (n  1)( n  2)
 1.75

Table value F(3,6) degrees of freedom 8.94


There is significant difference between treatments

14

Page no 85
Sathyabama Institute of Science and Technology

15

Page no 86
Sathyabama Institute of Science and Technology

STATISTICAL QUALITY CONTROL


Statistical quality control:
statistical quality control is a statistical method for finding whether the variation in the quality of the product is
due to random causes or assignable causes.
Objectives of statistical quality control:
To achieve better utilization of raw materials, to control waste and scrap and to optimize the quality of the
product without any defects.
Control chart:
It is a useful graphical method to find whether a process is in statistical quality control.
Uses of Quality control chart:
It helps in determining whether the goal set is being achieved by finding out whether the Process is in control or
not.
Different types of control chart:
Control chart for variables – Range and mean chart, Control chart for attributes- p-chart,
C-chart, np-chart.
control limits for mean chart:

Central limit = x , upper control limit = x +A2 R , lower control limit = x -A2 R

Where x is the mean of the sample and R is the range.


The control limits for range chart:

CL= R , UCL = D4 R , LCL=D3 R .

Procedure to draw the x  chart & R-chart:

1. The sample values in each of the N samples each of size ‘n’ will be given. Let X1, X 2 ,... X N be the means of the
N samples & R1, R2… RN be the ranges of the N samples.

2. Compute X 
1
N
  1
X 1  X 2  ...  X N ; R   R1  R2  ...  RN 
N

3. The values of A2,D3,D4 for the given sample size n are taken from the table of control chart constants.

4. Find the values of the control limits x  A 2 R ( for the mean chart) and the control limits D3 R and D 4 R ( for
the range chart) are computed.
5. On the ordinary graph sheet, the sample numbers are represented on the x-axis and the sample means on the

Page no 87
Sathyabama Institute of Science and Technology
y-axis ( for the mean chart) and the sample ranges on the y-axis(for the range chart).

6.For drawing the mean chart, we draw the three lines y  X , y  X  A2 R and y  X  A2 R which represent
respectively the central line, the L.C.L line and U.C.L line, Also we plot the points whose coordinates are
1, X , 2, X ,...N, X  and join adjacent points by line segments. The graph thus obtained is the X chart.
1 2 N

7. For drawing the mean chart, we draw the three lines y  R , y  D3 R and y  D4 R which represent respectively
the central line, the L.C.L line and U.C.L line, Also we plot the points whose coordinates are
1, R1 , 2, R2 ,...N , RN  and join adjacent points by line segments. The graph thus obtained is the R chart.

Mean And Range Chart Problems


1. Given below are the values of sample mean X and sample range R for 10 samples, each of size 5.
Draw the appropriate mean and range charts and comment on the state of control on the state of control
of the process.

Sample 1 2 3 4 5 6 7 8 9 10
No.
Mean X i 43 49 37 44 45 37 51 46 43 47

Range Ri 5 6 5 7 7 4 8 6 4 6

Solution:
1
X 
N
 Xi
 43  49  37  44  45  37  51  46  43  47
1
10
 44.2

1
R
N
 Ri
 5  6  5  7  7  4  8  6  4  6
1
10
 5.8

From the table of control chart for sample size n=5, we have A2  0.577, D3  0 & D4  2.115

i) Control limits for X chart:


CL (central line) = X  44.2

Page no 88
Sathyabama Institute of Science and Technology
LCL  X  A2 R  44.2  (0.577 )(5.8)  40.85

UCL  X  A2 R  44.2  (0.577 )(5.8)  47.55

Conclusion :

Since 2nd,3rd,6thand 7th sample means fall outside the control limits the statistical process is out of control
according to X chart

MEAN CHART
54

52 (7, 51)

50 (2, 49)

48 (10,…
(8,…
46 (5, 45)
(4, 44)
44 (9,…
(1, 43)
42

40

38
(3,… (6, 37)
36
0 1 2 3 4 5 6 7 8 9 10

ii) Control limits for R-Chart:


CL  R  5.8; LCL  D3 ; R  0
LCL  D4 R   2.115 5.8  12.267  12.27

Page no 89
Sathyabama Institute of Science and Technology
R Chart
13

12

11

10

8 (7, 8)
(4, 7)
7 (5, 7)
(10, 6)
6 (2, 6) (8, 6)
5 (1, 5) (3, 5)
4 (6, 4) (9, 4)
3

1
0 1 2 3 4 5 6 7 8 9 10

Conclusion :

Since all the sample means fall within the control limits the statistical process is under control according to R chart .

2. The following data give the measurements of 10 samples each of size 5 in the production process taken in
an interval of 2 hours. Calculate the sample means and ranges and draw the control charts for mean and
range.

Sample No. 1 2 3 4 5 6 7 8 9 10
Observed 49 50 50 48 47 52 49 55 53 54
measuremen
55 51 53 53 49 55 49 55 50 54
ts X
54 53 48 51 50 47 49 50 54 52
49 46 52 50 44 56 53 53 47 54
53 50 47 53 45 50 45 57 51 56
Solution:
1
X   Xi
N
1
 52  50  50  51  47  52  49  54  51  54 
10
 51.0

Page no 90
Sathyabama Institute of Science and Technology
1
R
N
 Ri
1
  6  7  6  5  6  9  8  7  7  4
10
 6.5
From the table of control chart for sample size n=5, we have A2  0.577, D3  0 & D4  2.115
i) Control limits for X chart:
CL (central line) = X  44.2
LCL  X  A2 R 2  51.0  (0.577)(6.5)  47.2495
UCL  X  A2 R 2  51.0  (0.577)(6.5)  54.7505
CL  X  51.0

Mean chart
55

54 (8, 54) (10, 54)

53
(6, 52)
52 (1, 52)

51 (4, 51) (9, 51)


(2, 50)
50
(3, 50)
49 (7, 49)

48

47 (5, 47)

46
0 1 2 3 4 5 6 7 8 9 10 11
Conclusion :
Since 5th sample mean fall outside the control limits the statistical process is out of control according to
X chart
ii) Control limits for R-Chart:
CL  R  6.5; LCL  D3 R  0
UCL  D4 R   2.115 6.5  13.7475

Page no 91
Sathyabama Institute of Science and Technology
Range chart

14

12

10
(6…
(7, 8)
8
(2, 7) (8, 7)
(9, 7)
(3…
6
(1, 6)
(5, 6)
(4, 5)
4 (10, 4)

2
0 1 2 3 4 5 6 7 8 9 10 11
Conclusion :
Since all the sample means fall within the control limits the statistical process is under control according to
R chart .

C-chart:
Control chart for number of defects is called c-chart.
The control limits for c-chart.

CL = c UCL = c +3 c LCL = c -3 c
C-Chart problems
1. 15 tape recorders were examined for quality control test. The number of defects in each tape recorder is
recorded below. Draw the appropriate control chart and comment on the state of control.

Unit No.(i) 1 2 3 4 5 6 7 8 9 10 11 12 13 14 15
No. of 2 4 3 1 1 2 5 3 6 7 3 1 4 2 1
defects (c)
Solution:
The number of defects per sample containing only one item is given,

Page no 92
Sathyabama Institute of Science and Technology
c
c  2  4  3 
i  2  1

45
3
N 15 15

CL  c  3; LCL  c  3 c  3  3 3   2.20

We take LCL  0 ( since LCL cannot be –ve)

UCL  c  3 c  3  3 3  8.20

c- chart
10
9
8
7 (10, 7)
6 (9, 6)
(7, 5)
5
(2, 4)
4 (13, 4)
(3,3)
3 (8, 3) (11, 3)
2 (1, 2) (14, 2)
(6,2)
1 (12, 1)
(4, 1) (5,1) (15, 1)
0
0 1 2 3 4 5 6 7 8 9 10 11 12 13 14 15 16

Since all the sample points lie within the LCL and UCL lines, the process is under control.

2. 20 pieces of cloth out of different rolls contained respectively 1,4,3,2,4,5,6,7,2,3,2,5,7,6,4,5,2,1,3 and 8

Page no 93
Sathyabama Institute of Science and Technology
imperfections.
Ascertain whether the process is in a state of statistical control.
Solution:
Let C denote the number of imperfections per unit.

c
Total no of defects

c
Total sample inspected n

1  4  3  2  4  5  ...  1  3  8
c =4
20

UCL= C  3 C  10

LCL= C  3 C  2

We take LCL  0 ( since LCL cannot be –ve)

c- chart
10

9
(20, 8)
8
(13, 7)
7 (8, 7)
(7, 6) (14, 6)
6

5 (6, 5) (16, 5)
(12, 5)
4 (2, 4) (5,4)
(10, 3) (15, 4)
3 (3, 3) (19, 3)

2 (4, 2)
(9, 2) (11, 2) (17, 2)
1 (1, 1) (18, 1)

0
0 1 2 3 4 5 6 7 8 9 10 11 12 13 14 15 16 17 18 19 20 21

Since all the sample points lie within the LCL and UCL lines, the process is under control.

Page no 94
Sathyabama Institute of Science and Technology

p-chart:
Control chart for fraction defectives is called p-chart.
control limits for p-chart.

UCL= np  3 npq , LCL= np  3 npq CL = np

np –chart.
Control chart for number of defectives is called np chart.

P-Chart & nP-Chart Problems


1. Construct a control chart for defectives for the following data:

Sample 1 2 3 4 5 6 7 8 9 10
No:
No. 90 65 85 70 80 80 70 95 90 75
inspected
:
No. of 9 7 3 2 9 5 3 9 6 7
defective
s:

Solution:
We note that the size of the simple varies from sample to sample. We can construct P-chart, provided 0.75 n < ni
<1.25 n , for all i.
Here
1 1
n
N
 ni  10  90  65  .....  90  75
1
 800   80
10

Hence The values of ni be between 60 and 100. Hence p-chart can be drawn by the method given below. Now p =
Total n o.of defectives
Total n o. of items inspected
60
=  0.075
800
Hence for the p-chart to be constructed,

Page no 95
Sathyabama Institute of Science and Technology
CL= p =0.075

LCL= p  3

p 1 p 
n
0.075 X 0.925
= 0.075  3  0.013
80
Since LCL cannot be negative,it is taken 0.

UCL= p  3
p 1 p  
n

0.075 X 0.925
= 0.075  3  0.163
80
The values of pi for the various samples are 0.100, 0.108, 0.035, 0.029, 0.113, 0.063, 0.043, 0.095, 0.067, 0.093

p-chart
0.17
0.16
0.15
0.14
0.13
0.12 (2, 0.11)
0.11 (5, 0.11)
0.1 (1, 0.1) (10, 0.09)
0.09 (8, 0.09)
0.08
0.07 (6, 0.06) (9, 0.07)
0.06
0.05
(3, 0.04)
0.04 (7, 0.04)
0.03 (4, 0.03)
0.02
0.01
0
0 1 2 3 4 5 6 7 8 9 10 11

Since all the sample points lie within the control lines, the process is under control.
2. The following are the figures for the number of defectives of 10 samples each containing 100 items
8,10,9,8,10,11,7,9,6,12 .Draw control chart for fraction defective and comment on the state of control of
the process.

10

Page no 96
Sathyabama Institute of Science and Technology
No.of defectivesin the sample
P for sample = 
No. of items in the sample

8
P for sample =  0.08
100

Similarly calculate p for each sample and tabulate.Divide the number of defectives by 100 to get the fraction
defective.
Sample 1 2 3 4 5 6 7 8 9 10
No:
No. of 8 10 9 8 10 11 7 9 6 12
defective
s:
P=fractio 0.08 0.10 0.09 0.08 0.10 0.11 0.07 0.09 0.06 0.12
n
defective
s

p
p
n
0.08  0.10  0.09  0.08  0.10  0.11  0.07  0.09  0.06  0.12
=  0.09
10

UCL= P  3

P 1 P 
n

0.09(0.91)
= 0.09  3  0.177
100

UCL= P  3

P 1 P 
n

0.09(0.91)
= 0.09  3  0.003
100

11

Page no 97
Sathyabama Institute of Science and Technology
0.18
p-Chart
0.16

0.14

0.12 (10, 0.12)


(6, 0.11)
(2, 0.1)
0.1 (5, 0.1)
(3, 0.09)
(8,0.09)
0.08 (1, 0.08) (4, 0.08)

(7, 0.07)
0.06 (9, 0.06)

0.04

0.02

0
0 2 4 6 8 10 12

Since all the sample points lie within the control lines, the process is under control.
3. The data given below are the number of defectives in 10 samples of 100 items each. Construct a p-chart
and an np-chart and comment on the results.

Sample No. 1 2 3 4 5 6 7 8 9 10
No. of 6 16 7 3 8 12 7 11 11 4
defectives
Solution:
Sample size is constant for all samples, n=100.
Total no. of defectives = 6 + 16+7+3+8+12+7+11+11+4= 85
Total no. Inspected= 10 x 100 = 1000

Total no.of defectives 85


Average fraction defective = p    0.085
Total no.of itemsinspected 1000

For p-chart:

p 1  p   0.085 (0.915)  0.0013


LCL  p  3  0.085  3
n 100

12

Page no 98
Sathyabama Institute of Science and Technology
p 1  p    0.085 0.915   0.1687
UCL  p  3  0.085  
n  3 
 

P CHART
0.18
(2, 0.16)
0.16

0.14
(6, 0.12)
0.12
(8,0.11)(9, 0.11)
0.1

0.08 (5, 0.08) (7, 0.07)


(3, 0.07)
0.06 (1, 0.06)

0.04 (10, 0.04)


(4, 0.03)
0.02

0
0 2 4 6 8 10

Conclusion:
All these values are less than UCL=0.1687 and greater than LCL=0.0013. In the control chart, all sample points
lie within the control limits. Hence, the process is under statistical control.
For np-chart:


UCL  n p  3 n p 1  p 


 n p3

p 1 p 
 
 n 
 

 100  0.1687   16.87

13

Page no 99
Sathyabama Institute of Science and Technology
np  100  0.085  8.5

LCL  n p  3 n p 1  p 


 n p 3

p 1 p 
 
 n 
 

 100  0.0013  0.13

np- chart
18
(2, 16)
16
14 (6, 12)
12
(8,11) (9, 11)
10
8 (5, 8) (7, 7)
(3, 7)
6 (1, 6)

4 (10, 4)
(4, 3)
2
0
0 2 4 6 8 10 12

Conclusion:
All the values of number of defectives in the table lie between 16.87 and 0.13. Hence, the process is under control
even in np-chart.

14

Page no 100

You might also like